You are on page 1of 218
Solutions Manual for MODERN ORGANIC SYNTHESIS: An Introduction Michael H. Nantz, University of Louisville Hasan Palandoken Western Kentucky University George S. Zwiefel University of California, Davis W.H. Freeman and Company ‘New York ISBN 13: 97807167-7494-5, ISBN 10: 0-7167-7494-1 (© 2006 by W. H, Freeman and Company All rights reserved Printed in the United Sates of America First printing W. H, Freeman and Company 41 Madison Avenue [New York, NY 10010 Houndaulls, Basingstoke RG21 GXS, England vwoywrwhfreeman.com Contents Preface iv CHAPTER. SYNTHETIC DESIGN Il CHAPTER 2, STEREOCHEMICAL CONSIDERATIONS IN PLANNING ‘SYNTHESES 2 CHAPTER 3. THE CONCEPT OF PROTECTING FUNCTIONAL GROUPS 40 CHAPTER 4. FUNCTIONAL GROUP TRANSFORMATIONS: OXIDATION AND REDUCTION 55 CHAPTER 5. FUNCTIONAL GROUP TRANSFORMATIONS: "THE CHEMISTRY OF CARDON-CARBON x-BONDS AND RELATED REACTIONS CHAPTER 6, FORMATION OF CARBON-CARBON SINGLE BONDS VIA ENOLATE ANIONS: 107 CHAPTER 7. FORMATION OF CARBON-CARBON BONDS VIA ORGANOMETALLIC REAGENTS 136 CHAPTER 8, FORMATION OF CARHON-CARHON #-BONDS 165 CHAPTER 9, SYNTHESES OF CARBOCYCLIC SYSTEMS 186 Preface How docs an organic chemist go about synthesizing a desired molecule? The goal of this Solutions Manual is not only to provide you with the “correct” answers to end-of- chapter problems, but also fo give you the opportunity to develop a methodical approach to symthesizing @ given target molecule using the tools and concepts covered in Modern Organic Synthesis: An Introduction. ‘We assume that th student is well acquainted with the basic concepts of organic chemistry taught atthe sophomore level, The introductory chapters provide step-by-step solutions to illustrate how a problem is broken down to smaller problems and solved. Later chapters assume the student is well versed in the concepts covered inthe previous chapters; however, the key steps involved in the solutions are highlighted. Throughout the manual, literature references are provided for problems that have been inspired by “real” world examples. ‘To assimilate synthetic methodologies and to integrate them into @ synthetic design requires actual problem solving and not merely looking at the text. Only by ‘writing the answer do we pay attention to details such as proper choice of reagents, ‘reaction conditions, mechanistic implications, and so forth. Thus its important that you refer tothe Solutions Manual only ater you have made a real effort to solve the problem, ‘or simply to verify that your approach is the same as or even Better than the one presented inthe manual. We are grateful to the Chemistry 131 students at UC Davis and especially 10 the tesching assistants of the course for their suggestions and contributions to the development of the problem sts. ‘Michael Nantz, Hasan Palandoken and George Zweifel, Tune 2006 ‘ABBREVIATIONS he. sey cate DMs ety ie Piven fo sic DMSO ollie PN tanta BN 2 nbioluroile DS. eet ety PTS oni fdas ce enatome ce lennon [Athenee “ “selene FP ci miro Bo eat IeProcomcl eT Boag EWG een viene ep fing er qa ih SUN Shoat ATmaRe FO fol pop rr Upbeat 9 Bn ent ‘ new Penn og atten i esi REN Raney ice (aly te. oti on MPN peameyipgponie Witgpe) BPs ciety HQ. Apogee REM ont oa ne) hy, ‘be ‘mete Bronce TX odoesbenele a Re Ant bi ett Indo amie (@-hosyeoxy) fr ten pe oceangeyt lenis die hneaile ADA pe * soon ergoiee B52 moteane ‘isowopylanie SAB Step ae Gin emplogentong KHIMDS satu ‘poscaon (COL enim eemetyide Stent eae Oncaea Leslie. ti SE rma yebony ©. ‘yeep ar ube ‘nett Ci ocamphlose wed LAK ont lamin ee ‘ineny ADCO. —naiee(222hetme LDA sapopytamise wi tyme DENA. LSdiaabeyel(4s0) LIMBS. Hien The onsen ee a DBL asoabaeabeyeS40)———ACPBA anaemia TES ehyla once Meson ma Thane Leiden MEM 2-neoryetboymetyl TFA tot i Dog Aetna mele TTEAA een aye itbenmanane MO. TH ey &, seer ens moni ot TP teanyoprn DEAD. at wnnetonyite Meno etanemleey) Thx. they CHM.) DET, et se MS. aoeear sees TS pron Ditty RaShgdo 2H pan MVE ami a one TDMEDA NDA sty IAD —liaopops emsoboqiae NDS. atremowecnne ‘edanine DIBA Hlieterysninmhyige NCS. ederim IMS noone IPT. aieprey tate RMON ‘MST ines DMAP -dimelamineyine Moxie ‘iowonebareatonse MDO—sinetsxiane NMP ent maiioe TRAP ot peymanion DME. aborgetiane OC, psi lero ‘een DMP Nslineigomeniée FDC. bom Tana igen) DMP. Dewi godine RG preci gp te tn (ptlenrly) DMPC NDP aimayeoplnees Phony 2 est CHAPTER 1 Synthetic Design Overview Chapter 1 focuses on how to design and execute the synthesis ofa variety of target molecules (TM) using the tools and strategies you encountered in ‘Modern Organic Synthesis: An Introduction. Problem 1 (Functional Group Interconversions) introduces how to recognize precursors of Key functional groups present in a target molecule. Problem 2 (Uimpoling) highlights the use of carbonyl group polarity reversal, an important synthetic tool. Problem 3 (Retrosyathetic Analysis) stresses approaches to disassembling a target molecule into simpler fragments and ultimately into starting materials. Problem 4 (Synthesis) provides the opportunity to put into practice these concepis by designing protocols for the syntheses of various target ‘molecules. Key Concepts Retrosynthetic analysis Synthetic equivalent (SE) Functional group interconversion (FGI) Carbony! group polarity reversal (Umpotung) 2 + Chapter Syatati Design SOLUTIONS TO CHAPTER 1 PROBLEMS ‘The more challenging problems are identified by an asterisk (*). 1, Functional Group Interconversion. Show how each ofthe following, ‘compounds can be prepared from the given starting material CXCH} OH —= EIO,C(CH.ICN Solution: Fis in this problem: 41. Primary (1) a choi to ety! ester 2.12 aloorol tonite Q 0° 4c), 1, DHP_, cat. T20H, by CHCHAOM Fe ES EIOACICH AON SEE EIOCICH:ACN 2.603 aso. 23, EI0H, cat, HeS0g Step 1 ‘The 1° alcohol is protected with dihydropyran (DHP). Step2a Reaction of an alkyl chloride with magnesium (Mg) provides the corresponding Grignard reagent (conversion of an electrophilic carbon to a nucleophile carbon). Step2b Reaction of a Grignard reagent with catbon dioxide affords the carboxslic acid Step3_ Fischer esterification (teaction ofa carboxylic acid with excess alcoho! and a strong acid catalyst) Step4 The 1° alcohol is converted to a good leaving group (losylate) for the ensuing displacement reaction (Step 5) Step 5 Sq2 displacement ofthe tosylate group with eyanide anion provides the ™. FI in this probe Carbonic ae to 1° ami Solon to Chapter Problems * 3 OW 200% C08 ay cre seo sure” CF ‘30. H, HzO S haot 0 Step 1 Carboxylic acid is converted to an acyl chloride with thionyl chloride (SOCI, or oxalyl ehoride ((CO).Ch}. Step2 Reaction ofthe acyl chloride with ammonia (NHs) affords the amide. Step 3a Reduction of the amide with lithium aluminum hydride (LiAIH,) provides the TM, Steps 3b+e Workup protocol O > Cha ms Solution: Fale in this problem: fi Alkane toast 2! Alene ta higher carbon homolog (2k 3. Unarcat, OH ‘Hp, EIOH, eat BOT ‘omy BY 2,94 NaHCO Step 1 Conversion ofthe alkene to the epoxide Step 2a Epoxide opening proceeds via Sx2 displacement to give the trans- adduct Step 2b Workup protocol Step 3 Semi-hydrogenation of the tiple bond with Lindlar’s catalyst gives the Qralkene. Step4 Conversion of the aleohol to the acetate with acetyl chloride (CH,C(O}CI or AcCh provides the TM. “ON = ONY 4+ Chapter Syatheie Design Solution: FI inthis problem: Che-carbon hemaogaton ofa terminal double bond to a terminal souorated carbon one ENB, Carnes, OO” Frecne” Orn.) “tee = OnWe 5 ‘Step 1 Hydroboration-oxidation yields the 1° aleaho Step2_ Oxidation with pyridinium chlorochromate (PCC) affords the aldehyde Step3 Wittig reaction provides the TM. (Note: The ylide reagent is prepared by treating PhyP with CDsI to obtain PhsP*CDy and followed by deprotonation with n-BuLi.) 2. alcoho to carboxylic a0 9 se Kn,THF,0°0 AL DHP cat Ht 3b. PhCH BF vm HOLTON gata HET HOY OTP “Trego oh Be NaOH HO bt Saaaton Step | 1° aleohol at C3) is protected as its tetrahydropyranyl (THP) ether, Step 2a Reduction of the carboxylic acid with lithium aluminum hydride (LiAIH, affords the aleobol. Step 2b Workup protacol. Stcp 3 Williamson ether synthesis provides the C(1) benzyl ether. Step4 THP hydrolysis at CG). Step 5 _ Jones oxidation produces the TM in 55% overall yield Reference: MeGuitk, . Collum, D.B. J. Org. Chem. 1984, 49, 843, Solutions to Caper 1 Problems + § 1a. UAH, THE 3.NalOs THRO aes ib. NaOH, HO, CR Rada MeOH, 0 “2M; MeOH 4.0105, H:80s H SSCS ON oy 6 & Gaon | — 6 a Lact (wemiaceta) Step 1a Reduction of the lactone with lithium aluminum hydride LiAIM, provides the diol Step tls Workup protocol Step 2 THP hydrolysis affords the trol with C(1), C(2) and C(4) bearing the hydroxyl groups. ‘Step 3 Oxidative cleavage of the 1,2-dio! (C(1)-C(2)] with sodium periodate (NalO,) gives the aldehyde at CQ). Step 4 Acidic media of Jones oxidation allows the in-situ formation of the lactol (a hemiacetal), whieh i then oxidized to the lactone (TM). 1 Fora similar synthesis, see Takano, S; Tamura, N.; Ogasavvar, K. Chem. Soc, Chem, Comms. 1981, 155. 2. Umpolung. Show how each of the following compounds can be prepared from the given starting material using either a formyl or an acyl anion equivalent in the synthetic scheme, wow —e (= 6 + Chapter 1 Sythe Desi Ja. mbubi, THR HS 2. Bul, THF, OD ESSE oA) See Swe yo Wer HO Step 1b Selective Sy2 displacement ofthe iodide (beter leaving group than chloride) Step 2. Intraolecl isplacement of the chloride Solution: on AAS AK scion uty 274 1 Weor Retype rescson ‘erage reaction iromatanopK = 10 Note that the deprotonation of the strong acid nitromethane (pK, = 10) is secomplished using the relatively weak base sodium methoxide (NaOMe), 9 CICHe(CHy}y CHC (CHL CH —P CHy(CHabCICH.dCt ‘Solution: Solaion to Chapter Prblens #7 ac uo eee IS Sher umes EIOH,quinotne ° Reference: For an analogous synthesis, see Smith, R. Gi; Daves, Jt, G. Ds Daterman, GE. J. Org. Chem. 1975, 40, 1593. “a. oo Solution: 2 ae als] Heche yea ‘OMe (.2-sston preva inbta atten EA) E rshanoyis ct wna ae eat 18, TMSCN, Znlp 2 winkd fr ioe A 1 eth coate re Af eof Chee ay te CE recente) ~~ Ca menonron ‘OH 4b. NaHCO, 9 secrets. MeOH Lx A cong |_HO AAA cote eet by " -eyanohydrin, unstable to base + Chapter 1 Synthetic Devon Step3b The resonance-stabilized enolate anion reacts both chemo- and regio- selectively: iodide displacement occurs in preference to 1,2-addition; alkylation occurs eto the nitile moiety Reference: For an analogous synthesis, ee Kang, 8. H; Lee, H.S. Tetrahedron Lett. 1998, 36, 6713, 3. Retrosynthetic Analysis — One-Step Disconnections. For each of the following compounds, suggest a one-step disconnection, Use FGIs as needed, Show charge patteras, the synthons, and the corresponding synthetic ‘equivalents * aay 3 (man Reroomtti eas ASS 5 APY. ayy, 0+ ot © Do tat ora at os ss Smet sou © Dy Bead | ND Sa, stone am Bi BEiee Lewis acid-eatalyzed (SnCl,) ring opening of the epoxide is regioselective, placing the nucleophile atthe more substituted carbon, Reference: Pearce, G. T; Gore, W. By Silverstein, R.M. J. Org. Chem, 1976, 41,2191. Solutions to Chaper | Problems * 9 ; aphe = y eee ° aphe avy hee iL whe eH HO (workup) v “ Me 5 are orm OF 3 7 + 2mengee Co mer Ma" OH > ‘MeO2C " ‘cyclohexene = Diels-Alder ransiom ‘The 3° allylic alcohol isnot well suited as a dienophile (bulky, acid-sensitive). A better choice is methyl acrylate (electron-leficieat alkene). Addition of MeMgBr to the ester afer cycloaddition Furishes the 3° aleohol moiety. 0 + Chapter Syotietic Design Synthesis: o le Me 4.meoee Dataterrescion (sHepineo! Conte wekos Me the = a singeral Retrosynthetic analysis: yen > “yt 8 acon sro “oo : : an Sint meer ar srs ar TH8 oo singel 50% Reference: Denniff, P; Macleod, 1 Whiting, D. A. J. Chem, Soe., Perkin Trans. 1981, 82, Soluions to Chaper | Prolews #11 Retrsyhetic ani: ° ° 4 hy an wy <“ Vv - E10" HIN, 2 >XE cos > ‘S o = s00S¢ An eres gu \Gebiainthe dosed cargo patom confuse sektion Synthesis ° B00 wok ae co keto ester ar condensation of two esters (Dieckmann condensation) ofthe ethyl ester provides the B-koto carboxylic acid ion of the carboxylic acid (Note: This requires a keto Step Inramote Step 2 Saponiia Step3_Decarboxy group) Retrosynthetie analysis: ° ° Fl COS 8 CO = CON ei : : 12 + Chapter 1 Syntstic Design ndered halide. Thus, the alkyation approach (shown below) isnot a sterically recommended. Care Oe, 4. Synthesis. Outline a retrosynthetc scheme for cach of the following target molecules using the indicated starting material. Show (I) the analysis (Gncluding FGI, synthons, and synthetic equivalents) and (2) the synthesis of each TM, oO” Retrosynhetc analysis: Foked or - Oe 1 3-chetone Loe, Ae) A Be) a owe on o u = BAN gy BN c0se) OMe One Dyse) Soluion to Chaper 1 Problems * 13 Synthesis @ ‘eile, gna OANA, on moe, y 2. Nal asoe, ow we Frcs rate rr Step 1 Phenol (pK,=10) deprotonation and alkylation gives B. Step2_ Conversion of the alkyl bromide to the corresponding alkyl iodide (Finkelstein reaction). Step3 Alkylation of the 1,3 ketone enolate A provides the TM. Reference: For a similar synthesis, soe Diana, G. D. Salvador, U, J Zalay, E. $3; Carabateas, P. M.; Williams, G. L} Collins, J. C. Med. Chem. 1971, 20, 757 ». ° wth © tom é én é o 8 Cpeyrenoptonn (ential coping) Retrosynthetc analysis: an we On = On ° age) . B > eAYyRewoo > RO Bonde GH * Reasons Unpatng "COnH ° a Cromer 4 LA + mmscticome <= LY Wow 8 we cise) ‘COR 14 + Chapter Syatatie Design Synthesis: 10. UDA THE 2.8. tower ey Sonor tS BA Xoo = 38 oH. MeOH ‘coon omar ome S89 4:66, cr > CHC A en MeOH, HO oY O<) Step 1a Deprotonation using the non-nucleophilc base LDA avoids 1,2-additon to thealdchyde C. Step 1b. Note that the -OH group of the alkyl iodide substrate A. must be protected prior to alkylation, Step2 Wittig reaction, Step3 Ester saponification and acidification, which also cleaves the acid- sensitive THP group. Step4 Head-o-til esterifications of two hydroxy acid substrates give the bis lactone. Step 5 Removal ofthe dthiane provides the TM. Reference: For an analogous synthesis, see Seebach, D.; Seuring, B.; Kalinowski, 1-O.; Lubosh, W; Renger, B. Angew. Chem. Int. Ed 1977, 16, 268 SOE om A Salwionsa Chapter | Pblens © 18 Retrosyntetic analysis: im_h0H + AKAD? Sn Eagon = formatdonye ro ee DR ONO ties u Me 2A on " 1 ral 2m E 2 ier econo se 8 Synthesise ta, nPrtighr (2 00) 2.Per, Ma Br £00 Bey owe TentnD Ae (wort) 2. Mg, YO Ess Ma oHOH 30.4" HO week Step3 The reaction ofa tertiary Grignard reagent witha hindered ketone may result in reduction andior enolizaton of the Ketone. However, in the present example, the use of formaldehyde (an excellent electrophile) circumvents these side reactions. DOA toe ey 16 + Chapter Syotetic Design Retrosynihetic analysis: ase) 266 Yrat Ye my . Ama Bay se ewe) Synths ta. tg £0 Oe 0 am, Bue A, Py 4 cleat 8 ee ™ Meo NN HME ton CS oN a of vatun (vanguitzet) Solations o Chapter 1 Problems + 17 Retrosynthetie analysis: ‘Tp canara am oa 1 or. a ee i or Ae DISE) EAS = electophitic Ph sracsisalt, —} as) oten cise) Synthesis: Nie 1.14010) iw 2eCATn, g BAL ay a zt Ho” baat > Ze naoHe 0 Step 1 Reaction of CH,C(O)CI with D affords the amide (weak activator compared to an amine), which should circumvent multiple substitutions in Step 2a. Step 2a. Elecirophilic aromatic substitution oceurs orth to the amide. Step 2b Workup. Step2c_ Hydrolysis ofthe amide provides B. Siep3 Reaction of the 2° amine (ofthe aromatic ring B) withthe ethyl ester forms the amide while the reaction ofthe 1° amine (of glycine A) with the carbonyl group forms the imine. Referene + For an analogous synthesis, see Gates, M. J. Org. Chem, 1980, 45, 1675, 18 + Chapter Syntbetie Design £ Synthesis: a ty Be LO] 2 cos 2 oe Pe 5 Hea TH wa © 73% 7 Hh 48. Bult ok 5, HgCla, Hg OH oO ‘OH EE won GH HO Step2 Conversion ofthe 1° alcohol othe 1° alkyl bromide. Reference: Sharma, A.; Iyer, P; Gamre, 8; Chattopadhyay, S. Synthesis 2004, 1037, Solon to Chapter 1 Poblame * 18 Ho . om OH (GHg,cr, 7° Retrosynthetic analysis: ne 4 FN on 2S GH CHacHy GH Gnayors I, gee ren ASE) ° (CHabeCHs, 69) Ure Fa HONS Eo” On Gdors (CHaWcls c Fat wit action) Ho HO NNO engPacticHeols e ° Synthesis: ‘pnporycHicy” a.taciiteg, gee! gPOHA(CH 60), eto nat dso i 3 MA ooo ~ ° Cowu, TH ® se HEH aoe HCN, H.0. Step 1 The diol E must be protected (acetal, in this example) for the Wittig reaction to proceed. The Wittig reagent is prepared from the ‘phosphonium jdide using sodium dimsylate, NaCH;S(O)CHS, Step2 Mild acid hydrolysis of the acetonide prevents acid-induced isomet- ization ofthe (Z)-alkene. Step3 Selective tosyltion ofthe 1° alcohol Step 4 DBU (1,8-diazabieyelo(S4.0}undee-7-ene), a bulky, non-nucleophilic nitrogen base, is used to form the epoxide, Step Sa Under basic conditions, the epoxide ring opening proceeds at the less substituted carbon (compare to Problem 38). Step Sb Workup protocol. Reference: Corey, E. 1; Kang, 1. J. Am. Chem, Soe. 981, 103, 4618, Ho. ho consider he 0 pes) ‘symmory 1 H Hom eg HOu! Fgl > > HO" H 1 be ase) 1. Dine . Wear teor sel au, of Pa Unig THF aha 2a. Etre A >I et XD see 1 MeOH, HzO. Step 2b+e Workup protocol Reference: For an analogous synthesis, see Khanapure, . P; Naja, N;; Manna, 8.3 Yang, J-I; Rokach, J. J Org. Chem. 1995, 60, 7548, Solutions © Chapter 1 Problems * 21 “Oo Retrosynthetic analysis: otros Unootng " sore f 7 mr + ee 2. Ager ‘eg (ea) Aion wot Sito a Sep! ideo psponacs oft ae wal eleton of tees (Homer-Wadsworth-Emmons modification of the Wittig reaction). Steps2+3. The euprate-mediatedI,4-addition and subsequent Lemicux-Johnson oxidation of a vinyl’ group are excellent procedures for the introduction of the f-formy/ group. Step4 Intramolecular aldol condensation provides the TM. Reference: For the synthetic sequence applied to a similar TM, see Corey, E. J; Smith 1G, J. Am. Chem. Soc. 1979, 101, 1038. CHAPTER 2 Stereochemical Considerations in Planning Syntheses Overview CChaptet 2 focuses on conformational analysis as a tool for assessing the relative reactivity and stereochemistry of eyelie compounds Problems 1-3 emphasize the three dimensional representation of various cyclic molecules and evaluation of their energies by the A, G, and U parameters In Problems 4-6, we apply conformational analysis to predict the reactivity of carbocyclic systems toward various reagents and to gather information regarding the preferred stereochemical course of the corresponding reactions. Further examples of applications of conformational analysis in organic synthesis are incomporated in Problems 7-9, Key Concepts + Conformational analysis + Corey and Feiner’s A, G, and U energy parameters + Al? strain Soltionse Chapter 2 Problems * 23 SOLUTIONS TO CHAPTER 2 PROBLEMS ‘The more challenging problems are identified by an asterisk (*. |. Draw the chair o the half-chair conformations (where applicable) for each of the molecules shown below and determine the corresponding E, and AE, values, Use the A, G, and U Values of Table 2.3 and assume 0.7 keal’mol for Mell Al2 strain Me ier A 0 Me A Me Mo Me “oH “we ir Me Me Me a b. © a Me, Me age Host 9 7 ‘OE Ho™ ‘ome s Me ws " Me "ite £ & Me 4 per H — w OF oy LD 4 Pr Me a 5 a 1 aie OH Inerstone: S| dO one 12-ciequatorial OH Pr fo 13nd bPr At Ep =Une+Uou+ 1/2 (Ane + Aon) + Are 184094 12.8409) "21 6.15 keal/mol Bi Ep = Gon + Gir =02 +08 = 1.0 kcalmol ABiy= 615-10 = 5.15 kcal/mol ter 2 Steeochemical Consideration in Planing Syatheses : oe ere = Me. : Me H Me ° wo 13-al Mo 1H Interactions: 213 1S-dantal Ke /H (cyclohexanone 5ystern) two 13-lanal 4 ‘ne 1:2dlauatorlal Me Mo Bp = Ane +2/3 (Ave) =18+28 (1.8) = 30 kealimal = Aint Gue* Gus =21F04 +08 = 29 kealimal ABD= 0-29 = 0.1 kealimol 7 oO Me ag te a a, 1 Mo Me ae Me Me 8 a ‘one AY? strain Me /H Interactions: two.A"? sain MoH ‘ne 1--caxial Me / Mo ALE = Une Ute +A? Melt = 1818407 =4.3 kealimol ABD = 43-14 = 29 kcal/mol Solution to Chaper2 Problems + 25 ‘We Me . = ow Me H Mer ue L , : eee eee econ acne: $0 lee SS rg A: Ep Inte * Ue 1/2 (Ame) pant eee B: Ep ~2(Gue+ Gy) + A'? Melt (04404) +07 =2.3 kealimol AB = 45-23 = 22 kealimol Ms ty] Ho: Me = 0° = Me Mé Ho a 2 1a Me OME erie Interactions: Oe 43lanal OMe fone {2cloquateial Me /OH ‘ne 1-donel OHH Une + ome + 1/2 (Arte + Aome + Aon) =18409 121840909) ' keal/mol AE Ep * Awe t Guet Gow = 18404402 4 keal/mol 45-24 = 21 keal/mol 26 + Chapter 2 SucsochemialConsideton in Planing Syntheses ; = = 5 ; CO = EAD = ee te aw "i mete get 4 ‘Me, Hs oo Me ; ; ee eeeeret A seam ee iandarann Si seneeoue Aust Une ncaa * 12 (Ane + Anca 18+ 18+ 184+ 12 (1.8 + 1.8) =72 kcal/mol AEp = 72-27 = 45 kealimol Solutions o Chapter? Problems © 27 Brace: fo 1, 3ladal CUt)-Mo Interactions: one 12-dequatrial CU -OH/CIB}CHy Bove: ‘wo 13-lawa O(1}.08174 one # 2dequatoria Ctaytte CiS}-CHe ‘ase + Gon * Gri on + Gs + Gans ‘ono 1,-ciawalC{6)-CHe/H fone 13-ciwalC(1}CHa seface: ‘wo 13-axal O(8)-CHy fone 13axal CCH, J Broa? V2 Anon, Brant 1 hn, ASE ~ Awe + Gow + Grey + Anes * 1/2 (Anca) = 184024044 18+ 12 (18) 5.1 kcal/mol Aon + Guue+ Great + Anca + 1/2 (Ancis) =09+04+04+18+12 (18) 44 keal/mol Ep AED= 51-44 28 + Chapter 2Stecochemicl Consideration in Planing Syatboses : = ee ae sca amas -diaxial C(2)-OEt/H ttre LOPY ot Tis + Uo #8 Ast 12 Roe Fhe ; , : ne ae ae Siler eetconremen Interactions: C1 Chip H ‘one 1,3-diaxial C(2}-O8t/ ‘one 1S! {8)-CHe/H ‘one 1 Scan Cs}-CH oes Unce.t 1 Roe #112 ncn 12 Ano Az Ep = Une + Uoer + 12 (Ay) + 12 (Aon) + Aner 18409412 (1.8) +1209) +18 85 keal/mol ite + Grex, + Uosr+ Unen, + 1/2 (Aon) + 1/2 (Anes) * 12 (Ancn,) 4+04+0.9+ 18+ 1/2 (0.9) + 1/2 (1.8) + 12(18) = 4.4 kealmol Ep ABD = 585~5.75 = 0.10 keal/mol Solutions to Chaper 2 Problems + 29 2, Draw the most stable conformation for each of the compounds shown below. You do not need to compute the E, and AF values. Solution: We A seo moo : ; oot, A? we ES " H » Me erates oH Me. 4 x We TH0 Sh mom == oO [ Notts i? a ‘OH Ht 4h Mon Bis the more stable conformer. 0 + Chapter 2 Stereochemiea Consideration in Planing Synthese ‘rans-antitrans; each rng is inthe chai conformation, a 8- b > a 5. Given below are the observed « :§ epoxide ratios from epoxidations of the 0,0-2cetals > §Sacetals + Deoxygenations «0 Satins Chap ens = A SOLUTIONS TO CHAPTER 3 PROBLEMS “The more challenging problems are Kentied by an asterisk (*. 1. Reagents. Give the ttre ofthe major products (A-G) expected rom the following reactions, Assume standard aqueous workup conditions are used for product isolation. ont. TBscL In, CeCe a DMF I8CIOR gone HO OH 2. NalOy, CHCl lege excons Solution: on 1. TBsaL ° imi, CeCe one -e-cen HO OH 2 Naldy CHCl ores A 0% Step 1 To obtain the monosilylation product, a large excess of the starting trol is used, Step 2. Vieinal diol cleavage. Reference: Paterson, 1; Delgado, O.; Florence, G. J; Lyothien, 1; O°Brien, Ms ‘Scott, Pz Sereinig, N. J. Org. Chem. 2005, 70, 150 ole 1. ToC py . gyg 2 HEGBH, THE Solution: Ce He O b. ‘Step 1. Conversion of the 2 aleoho! to the comesponding tosylate (a good leaving group), Step 2 Displacement of the tosylate with a hydride asthe nucleophile. (Note: Steps + 2 representa common deoxygenation procedure for alcohots,) 42 + Chapter 3 The Coney of Protecting Fnstiona Grows moe Pay Bialtee gy sume ‘oF 2,6-utidine, CHCl oe id ae Meosems, (OH NL Blige MeO:0%,_,OTBS Ch gee Ce ‘o" 2,6-tutidine, CHCly ‘Oo 2. UBH,, THE — ores 82% imi ‘Oo cd & ‘Step 1 Protection ofthe 1° alcohol in the presence of a 2° alcohol Step 2 Protection of the sterically hindered 2° alcohol requires the use of the reactive silylating reagent TBSOTT, Step 3. The milder reducing agent LiBH, was used for the reduction ofthe ester ‘moiety instead ofthe more powerful LiAIH., presumably to facilitate the ‘workup inthe presence of the aid-lbile OTr group. Step 4 Conversion ofthe resultant 1° alcohol to the corresponding ioe, Reference: Gu, Snider, B.B. Org. Lett 2003, 5, 4385, 1. HocHCHOH eat TeOH ° HWNoony Fe crise oar eo 2. sq NaHcOs Sa, Nad THF. 0°C 3b. Bn, et nuh 1. HoorecHoH CO Sette Doom Be cnyugor oa, Ook sateeoy MO 3a. NaH, THF, 0°C e orca nun Step 2b Workup protocol Soluios to Chaper Problems + 43 1. mBUNF fs) THE.O"Cton 2.(Mo0},C0~ 4. POC, CHCl, Po. cat Tn 8 PngPachesaoy 1850... Koon —aetret__g, move, NW 3. He, cat. POOH)YS 89% 6. Hp, Cal. POOH} IC i Seera oer co nau wane apace ea No 000, 950A cnn ett DT oy a hy 0 ee Oa , Boc EIOAa, st Boc fi Es P spnpeciowtin 0 * hy meee LO Gute be TE 7 r Le iS aria aor ~Y “eome Caos boo 2 aot Step 1 Deprotection of the silyl ethers, Step2 Acetonide formation Step3 Benzyl ether deprotection Step 4 PCC oxidation ofthe 1° alcohol should furnish the aldehyde; however, the authors reference below) used the Swem oxidation, Step 5 Wittig reaction. Step 6 Alkene hydrogenation (Note: Boe, -butoxycarbonyl, protecting group is stable tothe reaction conditions in Steps 1-6,) Reference: Wang, Qi; Sasaki, N. A. J. Org. Chem. 2008, 69, 4767. “#4 + Chapter The Concee of Protecting Funtonl Groups ‘ ' ‘reson, 2ettane runo_ ar 0A ae oie X 7 900, pt Toa CHCl 8. PCC, CHCl Sotuton: : 1-Tesom, 264 pmo A, Che wih cote Stee ‘come 2000, pA bate bu or bres se 9. FC. Cheb SRR ‘cOnMe 8 bres F Step I Silylation ofthe 2° alcohol Step 2 Removal of p-methoxybenzyl PMB) ether with 23-dichloro-5,6dicyano- Ind-benzoguinone (DDQ) requires 10 (present in pil 7 ue). Step 3 PCC oxidation ofthe I alcohol tothe corresponding aldehyde, Reference: Paterson, L; Delgado, 0.; Florence, G. J; Lyothier IL; O'Brien, Mz Scot, J.P Sereinig, N. J. Org. Chem. 2008, 70, 150. & 1, POC, CHCl, 0 °C Heat eaters ee ae « a TERE bao, coe ements eee se cere “A ee ee Ne ee Oe a s CHCly pers : oot Solutions to Chaper3 Problems * 45 ‘Step 5 Selective cleavage of the sterically less hindered TBS ether, Step 6 Conversion ofthe I° aleohol tothe corresponding iid. Reference: Nomura, L; Mukai, C. J. Org. Chem. 2004, 69,1803, 2. Selectivity. Show the product(s) expected forthe following transforma tions. 7 Me Me (Meo},CMoe Maho" ‘at T0H et ome on Solution: ‘ome Me Me (We0)-0Me2 Me Me 1 “eat. Ys0H4 Moz -bor WoL Mr HO nc (0%) on S Acetal formation occurs between the cis C(2)- and C(3)-OH groups rather than between the rans C(3)-and C(4)-OH groups. Reference: Gyergyei, Ks Toth, A Bajza, Ls Liptik, A. Synlett 1998, 127 ». ta.Mel 20a) the ore 4b, BPS-C1 (1.3 eq) 2. MsCl, EIN (xs), me ar onal iceqnncr Bt, orca ore” 82 ohare) 46 + Chapter 3 The Concept of Potting Functional Groups Solution: ta mau 200 ORS Ho6y Me 10. BPS: (09 east tN 8) 4 7 iid, OMF ‘oH wae NO) Teeanwnc Toe Oe 4 a bers 8: m — in bees # bees mest 22 & ‘Sep le Workup protocol. Slep2 Elimination of the mesylate group, Reference: Defosseux, M.; Blanchard, N.; Meyer, Cs Cossy, J. J. Org, Chem toon gens 024 tno 2705111260 , “eine” Slag Eaten nae” SE, OH (H20) DMF,0°C Sottion: ° Hoyo apse O. _sTs0H zea, d Aon en” Hor On EWN, reso” oH On om ‘cal. mBugNI fon ‘MF. O"C c2 rewoveral Step 1 Laetonization occurs between the carboxylic acid and the -OH that is Solutions to Chaper3 Problems « 47 Step 2 Selective silylation ofthe 2°-equatorial “OH group (reaction rate: 2°eq > 2°ax > 3°); the ammonium iodide presumably produces Ube more reactive TBS+1 species, Reference: Setkow, M; Kelling, As Schilde, U. Bur. J. Org. Chem. 2001, 2735. a 1. HC(OED CHO ca NHAOy, 4, 3. FBUOK, DMSO, 7B HghNHe “HO,cCOH THE HO Solution: cn HOI ae 3.0K cHo ° 7 ZHANM, 4 “EHO;CCOH MeOH THE, HO ot ba Stcp 1 Mild conditions forthe selective acetalization ofthe aldehyie Step2_ Hydrazone formation. Step 3 Wolfl-Kisehner reduction (Huang-Minlong modification) Step 4 Acetal hydrolysis. J.MoNta (oo seven) 3a. LAM TH No MeMieinosohert, Se. UA THE gp BTGU0 6a, o7 — eiy SD MOMHO She ‘Solution: ° 1. Mati (no solvent OHO Se. LIAM,, THE Metta ro ote Mo Se uate Tt Jp seen oN Ay gor rd Ho" OMe et oe on To ALA Me ow & ter 3 The Coney of Proteting Functional Groups Step 1 Lactone cleavage furnishes the amide. Step2_ Selective protection ofthe resultant 1° alcohol. Slop 3b. Workup protocol, Reference: Flla-Menye, J-R.; Sharma, V; Wang, G. J. Org. Chem. 2008, 70, me t ene ei rescnea a eee oS Or" Scot Zegna ae, Sobers amuinzey ee eee tesa eae eee ae Or Sen OO SS cm = 2 hehe, : more Fe a Sep 4 1,2-Addition of MeLi occurs from the less hindered face of the 5- membered ring Reference: Atanu Roy, A. Schneller, 8. W. J. Org. Chem. 2003, 68, 9269. tenes Tieo-c 8.7860, mia ey Teanecaase Oo ES or —2PANELCHOR gy ‘zac acOER” "Tip psc, BOR 55 PO, cH.Ci, Solions to Chaper3 Problems + 49 Solution ta Nat Whore (tle 9.T8SO cat. i, Ho Ox 1b. BB C iPraNER, CHeCle Eweo, "on “THe Pac, 208 YY “ores Bi ameSA Bgg SPC. Cree y fron 68n _antépeale opening 3. Retrosynthetic analysis. Outline a rtrosynthetc scheme forthe following target molecules, Show (1) the analysis (including FGI, syathons, synthetic ‘equivalents) and @) the synthesis of each TM. You may only use ‘compounds with five or fewer carbons as starting materials wie of, rot Retrorymthtc analysis: on we OPA 3 a eriaeta ° 9 2 meager ase) CH > . Qn i ° ‘+CHt == HCOED 368 Symes nwoonscron [4 mrro(o0e (ose) ToOH (ca), Cele Mgbe] 26a GT nce Fane | He ™ 0 + Chapter 3 The Concept of Potting Functional Gros Step] Protection of the keto grovp. Step2a Formation of the Grignard reagent (SE A), Step 2b Utilization of 0.5 equivalent of ethyl formate HC(O)OEt ensures thatthe SE A and Bare fonmed in a2: | ratio, a prerequisite forthe formation ofthe symmetical dketon, Step 2e Dilute HCI deprotects the keto groups and facilitates the formation of the hhemiacetal moiety in the TM, ed ID SD SD u A= w™t TS Ae) Synthesis: ge Q ro, Bryn CHs ~o ana, Bye spor Be "> BH 5 ee ° Sum jo aM =, Tien acetone re Step3_ THPO-CH,CHs-Br was used instead of ethylene oxide asa SE of A in the original procedure. Sotions to Chaper3 Problems * $1 Magatti, C. V.; Kaminski, 1.1; Rothberg, 1. J. Org. Chem. 1991, 56, 3102, Synthesis. Supply the missing reagents requited to accomplish each of the following transformations. Be sue to control the relative stereochemistry. - CH CHy Seq che Ho" GE oowt wo ye ey oh, Solution: ‘cal. TSOH 0 _HeoNets ZHOICHA-On Co FeO Ho(OMels gg CD monger oO (ea) c0aMe EO Step 3 Fischer esterification (H*, MeOH) would cleave the acetal protecting eroups. Step > Acetal hydrolysis followed by dehydration of the 3° alcohol furnishes the TM. Solution: wwe ot, f : y . dis See CLS Step 1 -acylation occurs preferentially over O-acylation, The amine must be protected to avoid N-oxidation in subsequent steps. Step 4 Dest-Martin oxidation was used inthe original procedure, Reference: libs, R; Bal, M; Busqu, Ps de March, P.; Elias, Ls Figueredo, M Font, J. Org. Lett 2004, 6, 1813. 2 vo Sooaty a 40-4. oon 1d ou oon Soluion WOH move Ae 2798009, ond wot eet Ae Ee Bae cre, ibe ou (Methyl a-D-mannopyranaside (OCH 4 4 x7) Anbu hon of BY tno se o- — °° [9 ——_»> m wo. mon To. SHO. ects cts Soluion to Chaper3 Problems + $3 Step 1 Selective formation ofa six-membered ring acetal using PhCHO. Step2._ Selective silylation ofthe equatorial hydroxyl group. ‘Step 5 Hydrolysis of bor the benzylic acetal and the glycoside ° ° oot om Solution: 2. HSCHCH.SH @ 1. HocHycHzoH cal BFyELO Seuon OH ay cas HSB) TNO, NaBH owe 4.5% ag HCL, THE H 9 resol OAR AAR we, Sotuion 160. ue NHy _-PrOH, ‘On On A Ho IS % owe 2.Twscl, THE TBSO. 3, Boz, EN TMS , Eons ‘TMSO aeene 6. KxCOs 180. TEthere OANA Ry ” ee o.,, On Teno ate Reference: Fora sinilar yess, ee Lannan, B.A Myers, AG. Org. Let 2004, 6, 104s. ‘54 + Chapter 3 The Concept of Protesting Functional Gros £ ° ° Hod Hao 0 0 cH p00 Hs Solution: © ue © np 4 2.108080 W HOLA saan tie MC Soren), P TENOR HOS yo PP? hexane, agp? on oe Gute om Step2 The acid-ctalyzed benzylation procedure prevents translactonization andior substrate polymerization *5.__Explain the regio- and stereochemical outcome ofthe following sequence of | reactions by showing the structures of the intermediates obtained after each step. NE " AE ‘a . eee be! Rom = poceen |e oot AR? bee tem ie Fe t it ney Product of Sep 1 Lewis alt assstedcloavage deste fhe aot sscueton 4 4 HoNA-O8_ s.prcHOvieln ttt Ho POP Hof ° 0 wn0% Step2_ Reductive cleavage ofthe acetal and reduction ofthe ester groups. Step3._ Selective wrans-acetalization of the 1,3-diol with benzaldehyde dimethyl acetal Reference: Cheol, E. L; Patk, Ms Yun, J.S. J. Am. Chem. Soe, 1998, 117, 017. CHAPTER 4 Functional Group Transformations: Oxidation and Reduction Overview Chapter 4 centers on two key tansfomations in organic synthesis: (1) cxidation of alcohols and of unsaturated hydrocarbon (i, alkenes and alkynes) {0 catbony] compounds; 2) reduction of various carbonyl compounds to alcohols Problem 1 emphasizes reagents for oxidation and reduction, Problems 2~ 4, stress the selectivity of oxidations and reductions in reaction sequences. Syntheses of Problem $ TMs require choosing specific reagents to achieve chemo, stereo, or enantioselective oxidations or reductions Key Concepts + Alcohol oxidations fo Jones, Swern, Dess-Martin ‘Chemoselective oxidations of allylic or benzylic alcohols (Oxidation of terminal alkynes Allylic oxidation of alkenes Reduction of carbonyl compounds © Nucleophilic reducing agents © Electrophilic reducing agents + Diasterooselective reduction of cyclic ketones ‘© Use of conformational analysis forthe prediction of hydride approach (ial vs equatorial) + Diastereoselective reduction of acyclic ketones > Cram’srule © Felkin-Anh model ‘+ Mitsunobu reaction (inversion of 2 alcohol stereochemistry) ‘+ Hydroxyl-dtected reduction of B-hydroxy ketones + Enantioselective reductions ter 4 Fanctooa Group Transformations: Ostion and Reston SOLUTIONS TO CHAPTER 4 PROBLEMS The more challenging problems are identified by an asterisk (*). 1. Reagents. Give the structure of the major products (AH) expected from the following reactions. Be sure to indicate product stereochemistries. ‘Assume that standard aqueous workup conditions are used for product isolation cos 1OMP.CH-O tt Fac 2 aN Me NaBH4CN, PPTS: MeOH Solution: ta EDP, CHeCosrt nebo Fe’ Bie NaBHCN, PETS: fonturamine MeOH (veuroacve dug) Step 1. Dess-Martin periodinane oxidation. Step2 Reductive amination ° 1. TBso1 THFsimid BOE, A DBALH Teg on £1,0,-78"to~torc 85% LLethy!lactate 2p, Rochate's sat ‘workup Solution: ° 1.Tesc 9 THE, ans wy (azeq wy on E40,-78°t-40"C —_OTBS Leettyliaciate 2, Rochole’s sat 8 workup cs Solion to Chaper 4 Problems 87 Step 2a Utilization of 1 eq of diisobutylaluminum hydride (DIBAL-H) ensures reduction ofthe ester group stops to the corresponding aldehyde Step 2b Workup protocol (Na-K tartrate) Reference: Defosseux, M.; Blanchard, N.; Meyer, C.; Cossy, J. J. Ong. Chem. on Ph OP 4. NaBHy ph OH CH:Clz, -78 °C Ph < “THE, MeOH bie -CO,H pute 3 Iniazele, OME, 2ametnye2-baore “Beinn! OS Sn ars cz oi ro Step 1 Note: Sodium borohytride (NaBH) can reduce lactones. Step2 Selective silylation ofthe least hindered 1° aleohol Slop 3 Swom oxidation ofthe 1° alcohol tothe aldehyde. Step 4 _ Oxidation ofthe aldehyde tothe corresponding carboxylic acid, (Note: Steps 3 +4 represent @ mild, stepwise oxidation of the 1° alcohol to a carboxylic acid.) Reference: Yamaguchi, K.; Kazuta, Vs Abe, H.; Matsuda, A. Shuto, 8. J. Org Chem. 2003, 68, 9255. one 1.0Me. aPso- CHsCle 3. PCC, NaOAe ~ tae gy SEER ON oe 7 BPhyP=CHy ym, OMe gay i (eq) io he eee Noto: The axcaes Wiig reagent loaves the acetate ester. ‘8 + Chapter 4 Functional Group Transformation: Oxidation and Redaction Solution: 1.DMP, BPso/" one Creo aPso~/” we 2 PhaPoCHe Ho (en ne THE ot ° a 3. PO0, Nao wo CHaCle he a cs Step 1 Dest-Martin oxidation. Reference: Jiang, L-; Martinelli, J. R Burke, S. D. J. Org. Chem. 2003, 68, 130) ° ey SPCC. CHO a: neMgh, EO 4.010, SO ‘CHZOH 25. HY, HO, Pat soetone, HO Solution: ' oO om CO. 1.707 cat. TS0H con Po. Ht Ho, mee aie "HOH a CHZOTHP Tet cho) Me Me co. 8. POG, CHCl CO. 4.0105, HeS0.™ ‘oHOH nt cou x ‘scot HeO Y Step3 PCC oxidation ofthe 1° alcohol tothe aldehyde. Step 4 _ Jones oxidation of the aldehyde to the corresponding carboxylic ac. (Note: Steps 3 +4 represent another two step oxidation of a 1° alcohol to the corresponding carboxylic acid: see Problem le.) Solutions to Chapter 4 Problems + 59 1. UAH, E40. 2. Mo,ClONe), cates, F CeCe joys product scat Tea bieyele product MO, CHyCN 12, LA, 40 2. Mez0(OMe, 1b. mild acid workup TBSO” cat. CSA, CHCl “2 samme oxo " ¥ 2% Step2 _Acetonide selectively produces the five-membered ring dioxolane Step 3 Oxidation of the I°alcahol tothe corresponding aldchyde. 4.4m, CHC om A HO On ‘OH cal TeOH, bonzone, * KOH, MeOH fi 4. P06, GhsCle 73% Solution: 4 wore CHCl, HO Ny wb ba eal, 30H, benzene, TKonMeoH CO 4 POG, CHsCly fi s 7% Step 1 Oxidation ofthe allylic alcohol tthe corresponding aldehyde. Step2 Protection of the resultant aldehyde as an acetal wed er 4 Fuser Group Transformations: Oidaton and Retin Reference: Tanis, S.P.; Nakanishi K, J. Am. Chem, Soc, 1979, 101, 4398. s 2. Snr wo Pecan TSCLINLOME eu OE Those 2 uan0r50), Teco HO EIOH, -78 °C Lad ee Sotton: 1. SOL iid, OF Ho heats OME e380 Yates NHBC 2. LIAH(OHBu)y_ NHBoo Boner ont ra oe 9 8 oi soci a ones Taso 4 oem cnn esata 9. Nat noe Satpsunt oui tH Hots Torcon” “OG, HO * be a Step2 _Chelation-controlled reduction of the Ketone produces the ant-aleobol dliasterooselectvely, Step 3 ‘The ammonium iodide salt presumably converts the BnBr tothe more reactive alkylating agent (Ba), Step 4 A basic workup is used to obtai the amino-sleohol proict. Reference: So, R. C.; Néonye, R.; lzmirian, D. P. Richardson, 8. K. Guerrara, RL; Howell, A. J. Org. Chem. 2004, 69, 3233, Solion to Chapter Problems * 61 2. Selectivity. Show the product(s) obtained or the appropriate reagent(s) to be used for the fllowing transformations, HQ one PO, wo steps one stop pore eee, 0% set écarbony proauct Solution: a oF Ho, ore 8 o tuso “orus 7%, 1.60. MooH (ed ay 2ONP 2509) eat THSOTT mo py. CHC, cHcl, -78 "°C Reference: Shi, B; Hawryluk, NA Snider, B. B. J. Org. Chem, 2003, 68, 1030, 1 Nabe 2 EIOH.0°C, LUA, THE SOO gy oe AW 088 ears CBr, Pha Solution: 1. Nay EIoH.o°c 9TH ° ALAA 0008 Ferrer” AN 0088 a Bt 0 Sa. UAH, THF 8b. mila acid wornup gee CBr, PPh Br 22 62 + Chapter 4 Functional Group Transformations: Oxidation and Redistion 7 ‘OH one ste, 0 H Ho" Solution: 0 0°, ‘0 OM pgcOscete, ‘atone Ho" Fotzots wagon HO" Alternatively, MnO: or Ba{MnOs]» may be used as selective reagents fr the ‘oxidation of lye or benzylic alcohols, a chr Hoo. 09 hon Heo en OH Solution: MoH on ong POC. che HO ce On Ob ‘The 2° axial aleobolis oxidized preferentially ve rl cb aa cb Ho" reso © 05% Soluions to Chaper4 Problems Solution: 1.TEMPO (eat) PhiOAe}e (CHCl, GHON Step 1. Chemoselective oxidation ofthe primary aleohol;(diacetoxyiodo)benzene (DIB) is the stoichiometric co-oxidant. Reference: Momin, E.; Nicolet, D; Mourifo, A. J. Org. Chem. 2004, 69, 4615. oa R 7 one: eee No. orlepewce OH ° homiacetl Solution: 220" (1, 26 bans) 2uon tHe} 2. OAD, hy ee Bae moa or ee on 0% Step! 12-reduston ofthe non fellowed by lactoniation, Steps2+3- Intramolecular Mitsunobu reseton, Stcp4 Controlled DIBAL-II reduction (Ie) tothe lactol Reference: Kang, S.H.; Lee, H.S. Tetrahedron Lett, 1995, 36,6713, 64 + Chapter 4 Function! Group Trssfomations: Oxidation and Reduction ang 2 1-H, POs CxO Tecmo i Steg Tueore | 7% ° Kssectise | gy 5% Solution. 2a Bly SMee HQ) THEO 4 2». E10H Step 2b Workup protocol where the reaction is quenched with EtOH and the resultant B(OEt) is evaporated. Reference: Lee, H. K; Chun, J. Chwang, . P. J Org. Chem, 2008, 68,2471. 3. Stercochemistry. Predict the stereochemistry of the major products formed (AGH) in the following reactions. Explain your choices. a LWMe THE side warp Sotions to Chaper Problems «65 Solution: om gh ‘onset of tet eX mea H we oa Feed mod! eH a Be Mee wd ho A ap 1. DIBAL-H (1.1 69) 7 roxane,-T6Cton 2 OCHOA i DEAD, PsP soot a. tan He Solution: coon heared "e Wo ra pH 1 (ited |e ve Terane comexiace | x ones i Fie "ase aoM 80% Ub THE 2. c1c0t4 o1o)oHeC! BB. NaOH HO oH Sena em oc we < Cn 8 586 for stops 243 Step 1 1,2-Addliton oocurs from the least hindered convex fac. 66 + Chapter 4 Functional Group Transformation: Oxidation an Resin ‘Step 2. Several procedures fr the Mitsunobu reaction were tried; the best results ‘were achieved using the more acidic CICH,CO Reference: Clive, D. L.1; Magnuson, S..; Manning, H. W.; Mayhew, D.L. J. (Org. Chem. 1996, 61,2095, The large steric requirement of L-Selectride generally favors reduction of ‘yclohexanones to form axial alcohols. However, inthis cis-decalone example, the formation of the axial alcohol is hindered by the adjacent ring. ‘owom NaBls, CoC, MeoHo°c ~ Solution to Chaper 4 Problems + 67 Solution: LoMoM owom Nabe C20, a Weonoe # ‘0 ‘OH Dees) ‘consider hoax antiparatel)adcton of hyde (= CHOMOM) ees Pie = = o ~ 4 @® ‘avored approach cistevores approach Note: Luche reagent (NaBH, CeCl) favors 1,2-additions. Reference: Mohr, P.J. Haleomb, R.L. J. Am. Chem. Soe. 2003, 125, 1712 Hac. 0 1, LUBHsBulp THE, _S.NACN,OMSO, 5 2Tslpy a8, UAH THE Sheol oan Sotatin: Hee. Cy ead H OTs HC, A OTS: OY aH nodes = OT x we 2. DMSO. HC... ! conn Sues ‘won 2) ‘Step 1 Equatorial delivery of hydride from L-Selectride. Step 3 Tosylate displacement occurs with an inversion of stereochemistry. Step 4 The amine (Ire base) i isolated aera basic workup, (6 + Chapter 4 Fonction Group Tansfomasions Oxidation an Retin oa 10. £1,80Me ‘e780 a.0BALH (2 He te Nastiy feena-76°6" AAA conten SSS py op 2 TSO 09), ini, OMAP eat) Ours Solution: stereocontol este FY resides equatorial yeti aac va axl ta enB0Me He (ntperaoh approach ieore-c Het Nabi . F Ade icous eu a cool Re et l= cHecHcHy = cH_00,¢8 2. TESCI 25 00) Ho on im, cat DMR fo. aq Nach a piecea euic conte syn 1 reso ores OBALMIA269) —te59 ores Coptan Ome OL cro 4 3 Rocheles sat y 0% 968, yn: an) Step Le Workup protocol Step 38, Workup protocol (Na-K tara) Reference: Fettes, As Carreira, E. M. J. Org. Chem. 2003, 68, 9274 *e tan | _NabHe jen | Ng MeOH Me Solution: o Reduction of the iminium ion proceeds via perpendicular hydride attack (nt jt.n |_NaBHs yi i MeOH Me, taco (Hy ' Soluion to Chap 4 Problems 62 face 1H) favored contormer - Na aul aa No CX arch.on fe OOH A%2otain “Me ci product e . aS 928 trans: os paralet) via a chair-lke TS minimizing A' strain Reference: Hofinan, 8; De Baccke, G.; Benoit, Ki De Clerg, P. J. Synthesis 1998, 479. “de. Ore Gort Solution: &- @ epinone {10 + Chapter 4 Functional Group Tansfomtions Oxidation and Reduction Workup procedure: ava a-pnene (vacuum) biacd eh0 ia HOCH,CR NH 4 ter precipitate Stepa_«-Finene isthe by-product formed nthe reduction. ‘Sep Diethanolamino i addad to sequester boron rom the bocon itermediates ‘Step d_Fltation removes the insoluble cethanclamine-boron complex. ‘The streocherrical outcome may be explained by considering the folowing ranstion Sale! ge} ot week (a A gemimethy points anay ‘rom the exs-CHy Reference: Brown, H. C.; Chandrasekharan, J; Ramachandran, P. V. J. Am. Chem Soc, 1988, 110, 1539, “1, Propose a method to accomplish the following stereochemical inversion, 9. 9: Rn R 0 — La ° ° Solution: 0 18.20% aq Neo 3. LSeletide § Meohtnt § Gog, THe, 1b NHC (workup) 4 “B10 09°C ° 2 DMP, CH;Cia ° Hose 4 “any ketone: use chelation conto! Soto to Chapter 4 Problems #71 4, PPTS, CcHe Pe rout 8 Ho, “10 79% overall ° Step 32.0 equivalents of L-Selectride are required since the carboxylic acid is ‘deprotonated on addition ofthe hydride reagent Referen: Yoshimitsu, T; Makino, T; Nagaoka, H. J. Org. Chem, 2004, 09, 1993, 4. Reactivity. Explain the regioselectivty and stereochemistry observe in the ‘transformations below. 1 aes0Me HO ~coyy cao %e BT noses th (crac se ict mune gee chat Solution: 1. MasOMe HO ~cogy HOHE ou sx 0. oo a YK & aw) AS |e Yoo intramolecular eancestricaon 75% Step 1 Acetonide formation occurs selectively to give the five-membered ring dioxolane. Step2_ Chemoselective reduction of COOH, Reference: Collum, D. 1980, 102, 2118. MeDonald, J. H; Sul, W. C.J. Am. Chem, Soe fr 4 Functional Group Transformations: Oxidation sn Reduction b. % NaBH CoCiy7 HO HO, ? Y Scho EIOH, HO ? Q ‘cio Solution: ° hydrate ont Ceci, promotes the formation ofthe hate: hence, FPOH —oniy th keto group isavaabie for educt Reference: Luche, J.-L; Gemal, A. L. J. Am, Chem. Soc. 1979, 101, 5848 omy o on J mono ene ine bb cron” SS 63 ' eco is th sa Solon: on aa oa 7 i PAHO caicaayes 69 RH HSS A ‘ace x "Smeton ‘oxocarbenivm fn ‘The initially formed oxocarbenium ion intermediate may be intercepted by the acetonide oxygens to form oxonium ion species (depicted below). - Acstonide rupture with eventual oss of acetone yields benzylidene acetals (eg, A and B), \whieh equilibrate via C to afford the axial hydroxymethyl product. Aiteratively, in the presence of the water formed, the acctonide may hydrolyze to provide & ‘more direct route C, Soluions to Chaper4 Problems + 73, on pha Howe. Pr, r 4-0. “ Pre "fl a Ne. + SA x Ey Ph Pn Ph c major product hygrogen bonding stabilized by Reference: Kang, 8. H; Kang, S. ¥; Kim, C. M.; Choi, H. Jun, H.-Y; Lee, B. Mz Park, C. M3 Jeong, J. W. ngew: Chem, Int. Ed. 2003, 42, 4779, “a. OMe opm OwALH oo TigO hexane Ans meres 14» Chapter 4 Functional Grup Teaformations Oridaton and Redostion Solution r= pmatoxphanyl ome methonypeny Wow aM oul g hen oder ioie otBAL Atstavores Hoar cy amie coxozarbeium fon ‘were | re toner Har Hs oPpMB wwothyp EYAL 'g MeOH apo 2% Reference: Holloway, G. As Hugel, HM. Rizzacasa, M. A. J. Org. Chem. 2003, 68, 2200, 5. Synthesis. Supply the missing reagents required to accomplish each of the following syntheses. Be sure to control the relative stereochemisty, Ho. ©. b wo Jen ° Solon to Chapter 4 Problems © 78 Solution: 1. Swern 1, 2 oO: ‘cat. p-TSOH, 4.4 HO OH 0 CHCl, rt Ho" 220% transacetalzation (steps 1-4) Reference: Su, Z; Paquette, L.A. J. Org. Chem. 1998, 60, 764 . an > oe ° 41, NaBH, CaCly, lad on won’ or 7 Ae0, EN Reference: Curran, D.P; Rakiewiez, D.M. J. Am. Chem. Soc. 1985, 107, 1448, ce oe Solution: higpae 40, 0BALH err Bees, 2. MeSO401(24 69) ‘orane, Oc™ asia a ore Tien a "ree SRS e on Ho ‘Step 3 Mesylate displacement oceus with inversion of stereochemistry Step 4 Acidic workup is required to hydrolize the intermediate imine that is formed on nitrile reduction Reference: McDermott, T. 8; Mortlock, A. A. Heatheock, C. H. J. Ong Chem. 1996, 6/, 700. 16 + Chapter 4 Functional Group Transformations: Oxidation and Reduction a on OH Ho" Ho Solution: 3a. LA, THE 1. Nal (9.069), QCIO}PH 3b. NaOH, HzO TOS-Ci, THE (workup) “2.DEAD, Pho? EO BUNE, THF PRCOH, THE Taso” 83% Step 1 NaH] treatment results in mono-alkoxide formation, which is critical to minimize formation ofa bis-silyl ether. In the original procedure, 96% ‘yield i reported for the formation ofthe mono-silyl ether. Referen: Clive, D.L. J; Magnuson, S. R; Manning, H.W, Mayhew, D.L. J (Org. Chem. 1996, 61,2095, on on wo, __ oh, Hom YY "ocHy Hoy “oots & bn Solution: 1. gc oS on on iN hee HOG, cat. DMAP Oo ou, 2 MozOIOWO, , io (OCs cat pTsoH ‘OCH on 6 BHM Ope on CHCl rt Step2 The cs 1,2-diol reacts selectively to form an acetonide, Step4 Both acetals (aceonide and glycoside) ate hydrolyzed as well as the trityl aroup. Step 5 is required to selectively e-form the glycoside Solutions o Chapter 4 Problems © 77 “ on © oe 12d oo4 oars Solution: 1. pts aides scat 1308 aes, mor on anepTOn, ok s:BPsc 1080, eisee Sono bum a fo uN ea MA 100 A\com TaBOR puwo7V° Sarr pur Earau puso 7.000 owe one CeCe, HzO aps Ste HO, foeral po ANS ose ON 42% overt yield 8: EN 20% Reference: Pattenden, G.; Plowright, A. Ts Tomas, J. Ax Ye, T. Tetrahedron Lett. 1998, 39, 6099. 9 ° wo\ on > won Ni NHBoe Solution: 4. (800).0) ° (12609), Nao Ho Yon Meo me Hel (ted 5. mBu,F, THF Step4 A minimum of two equvelents of DIBAL-H is required due to the presence of the active N-H, Reference: Nicolaou, K. C5 Bunnage, M. E; Koide, K, J. Am. Chem. Soc. 1994, 116, 402. 178 + Chapter 4Funionat Group Transformations Oiation and Reston B10,0. a ‘coMe ‘CH2OH ‘Solutions 300.8 1.oF,c04s ] oe ne Oya ZansTHF (Lea A 5 THF, ~10 °C tort a 2% 3. TBS ini, OMe T3807 i A E40,0°C tort Goad 4b. sat'aq Nap, 86% (worn) oh, Step 1 Selective cleavage of the Bu ester (analogous to Boe deprotection), Step2 _Chemoselective reduction of a carboxylic acid in the presence of an ster Reference: Dauben, W. .; Warshawsky, A. M. J. Org, Chem. 1990, 55, 3075. or wh Sotuion atom -Y° 3. soci, ety SL cout SMe ay Zuse. = Gary THF avers we eins ‘step ‘uo Step 1 Preferential formation of the six-membered acetal rather thaa the seven membered acetal Step 6 Basie workup afler acetal hydrolysis to obtain the amine in is free base form. CHAPTER 5 Functional Group Transformations: ‘The Chemistry of Carbon-Carbon 1-Bonds and Related Reactions Overview Chapter 5 deals with transformations of carbon-carbon r-bonds into a variety of functional groups. Problem 1 highlights reagents for carbon-carbon x-system transforms tions, Problems 2-4 and 6 emphasize selectivity in reactions of carbon-carbon m- bonds, The syntheses of TMs in Problems 5 and 7 require the selection of specifi reagents to achieve chemo, stereo, or enantoselectivty Key Concepts + Hydrogenation of alkenes + Dissolving metal reductions Birch reduction ion of alkenes ‘2. Hydroboration-oxidation of alkenes ‘9. Oxymercuration-demercuration + Epoxidation of alkenes ‘+ Epoxidatio of allylic alcohols ‘0 Sharpless asymmetic epoxidation + Dihydroxylation of alkenes ‘9. Sharpless asymmetric dihydroxylation + Hatolactonization + Cleavage of carbon © Ozonolysis © Lemieux-Jobnson oxidation + Somireduction of alkynes ‘© Conversion af alkynes to ( © Conversion of alkynes to (Z)-alkenes + Hydration of alkynes yon double bonds ~ ptr 5 Functional Group Transformations: The Chemisty of Cabor-Catbon x Bonds SOLUTIONS TO CHAPTER 5 PROBLEMS ‘The more challenging problems are identified by an asterisk (*). 1, Reagents. Give the structures of the major product(s) expected for each of the following reactions, Be sure to indicate product stereochemistry where More Zeon AY to HN CHs |= ES HN. UCHs oworonen 2 iy + os i a cs Reference: Brown, H, C; Kurek, 1.7. J. Am. Chem. Soc. 1969, 9, 5647, », 1.Na 25.90), NH () oxyene E20. 81H ey 22 SHe THE. Za. cat (PrepaARE “Gb. NAOH, HDs tolvene Solution: anaes cry HMB ese 5 D ow 2.Ha (PhP ANC ae ‘Hy 8 Step2__ Wilkinson's eatalyst allows for the selective hydrogenation of theless substituted double bond, Solon to Chap 5 Problems + 81 Gis .ncreA Ghd o-c Ba LOA G29) TERE gg aH 2 manic! "Hy en) Solution: ot 9 He Of aa torgecd 410, 1.necP6n nF Ro Ghee Bahl vc wc s nec oH, os oH je ¢ a Se ‘Step 1 Stereosclective epoxidation from theless hindered «face. Slep2 _ Regioselective base-modiated epoxide elimination. Reference: Malkov, A. V.; Pemazza, D.; Bell, M; Bella, M.; Massa, A.; Teply, Fs Meghani,P; Kocovsky,P. J. Org. Chem. 2003, 68, 4727. OMOM 3, DMOO.CH.C, 8. TMSOL(1 ec) acetone ELN, CHCl 2 KOH HO Baie EMOMGL FPN gory DMSO, ¥20 0 DMAP, CH,Cl 5. TBAF, THE, O°C Solution: 1. M00, 2. KOH, HO Mom “CHeCin ‘wom DMSO, oom oS yr ® Step 1 Exo (convex) face approach of oxidant gives B-face epoxide Hof Dt ah 82 + Chapter $ Functional Group Tnsfomatons: The Chemistry of Carbon.Cuton Bonds 8. TMSCI(1 eq) POM 4. wromos, Hert pom E4N,CH.Cle DMAP, CHaCle _ TSAR, THF, 0°C ase O” ne owou be Bee Referen Lepage, 0. Deslongchamps, P. J. Org. Chem. 2003, 68, 2183. OMe 1.UL NH) Be ikon a meron cto4 06 Toueoan” E Tea te wee Cofouae Shon. aro Sotto: om Lun mg0 Bt on 2. nce cH 0-0 oe imo MeoHHO cal DMAP, CHO t Me ery Ge es Ps & a Step 2 Mild acid catalyzes the enol ether hydrolysis without subsequent fy 9 4, B alkene isomerization. Step 4 Hunig’s base promotes P-climination ofthe epoxide followed by O-acylation, References Piers, E; Oballa, R.M. J. Org. Chem. 1996, 61, 8438, © tig natcos fauo Cont 5 tBuOK TAP “30505 cat), I Een shear HS Solutions to Chap Problems + 83 1 gk NaHOOS 06H 20) fat 60H 2 bBUOK.THF ore" 3.030. (a), NO 7 UGH, 150 i 4.2q, NaOH oO 1 : ‘ng formation 's pretemed (art’praie! opening ‘tthe odonim an) ® 18, Slg8H, THE 1b. NaOH, Oe 2 HalOAg, THR FO © 8. NaBHy NaOH, HO ‘Solution: 18, 89,64, THE IELNGH Oe “ZHgOAc THE, HO 3.Naaity NaOH HO Step 1 Hydroboration-oxidation of sterically less hindered terminal alkene. Step2_ Oxymereuration-demercuration of the intemal alkene yields the diol G. 40. NH (0 1B ag NHAC! work 1 B mCPBA CHOe HO workup 12, Na, NHS (9 1b, ag NYCI 2a, quic: workup 9 THE Ne a ZmePBA CHR Bu“ HO wore Ht 80 Hee 1. mcPBA Hae Gyconexane Raaye ae 2 KO, (12 ea) A MeOH a eso 3, POC, CHsCle Solutio Hee 1. moran Hee eyeiohexane tee PH 9.p00. re 2 mraye Heche 216605 (1.268) > a Mot f a 94% 1 Te80 Steps 1-2 Baeyer-Villiger oxidation followed by acetate hydrolysis, Reference: Momin, E.; Nicoletti,D.; Mourio, A. J. Org. Chem. 2004, 69, 4615, x 12. Na, Ns E10 4 coh BRA om, 1 an NHC! workup Solutions to Chapter $ Problems + 88 we ‘12, Na, NHs (0), we Chay, Sa te. aq NHC! = worn 4 ait Reference: Chuang, CP. Hart,D.. J. Org. Chem. 1983, 48, 1782, ; whe me & © OMe (CF yCO}0_ om 2 ea Py i : a er ee O Ome — (CF3CO},0 OMe a rw ‘ ae f & Step 1 Diaxial opening ofthe epoxide. Siep2 Modified Swern oxidation; see J. Org. Chem. 1998, 63, 8522. Step3 Hydride addition occurs preferentially from the a-face (and to the axiat isopropeny! moiety). Reference: Chen, L-; Wiemer, D.F. J. Org. Chem. 2002, 67, 7561 lr $ Fenton Group Transformitions: The Chemisty of Csbon-Cabon Bonds 2. Selectivity. Show the product(s) obtained or the appropriate reagents) to be used forthe following transformations, 0. 1-0 aH BAceO. pyran > ‘Tea 080, Naiog (excess) 1,0, MeoH CHa, E10 Hose Solution: 0. 4. CHa £0 jc _2.Aez0. pine at. 020, Nios (exesss) HOC” OH 10, MeOH Step2Lemieux-Johnson oxidation, Reference: For an analogous synthesis, see Woodward, R. B.; Bickel, B. H.; Frey, A.J; Kierstead, R, W. Tetrahedron 1988, 2, |. Solon to Chap Problems © 87 Soto: ssa on ony BBKOHCSA on y 1S Siee y 2m J was who WA AS 76% on oe 98.05 moot Chich-70°0 «GH MED Hog Sis Me bx mod Oa B.ere sexo Reference: Roush, W. R: Brown, R.J. J. Org. Chem. 1983, 48, 5093. = Be {_pon ‘ORE (one-pot) 58% Solution: a. Siag0H, THE BCHICHCOH “ENGOH, 302 = ‘one 50% ‘Step b _ Protonolysis of the intermediate vinylborane Step¢ Oxidation ofthe Sia:8- moieties as well as saponification ofthe acetate. (Note: Steps e-c represent a method to prepare Z-alkenes without the use of Hin the presence of Lindlar's catalyst Reference: Corey, E.; Herron, D.K, Tetrahedron Let. 1971, 1641 4 1.020 (at) 10, pyridine (2:1) sae > 2 Nal, H0,THF 3.Ag0H,0,THF SO a 1.0804 (ca) E10, pyriine @: 1) sree p Bao. ryo Tar ~ Howe OD fi a A 3.4920. Step 1 Dihydroxylation ofthe terminal alkene. Sicp2 Oxidative cleavage of the resultant 1,2 Step3 Oxidation ofthe resultant aldehyde to the corresponding carboxylic acid, Reference: Nozoe, S 1976, 195. Furukawa, J; Sankawa, U.; Shibata, S. Tetrahedron Let, 4 cD 2 ee B.NEOHH:02 major minor te pp ape we t t b waa bo 4 a " ae) ; : The hydraboration occurs from the ess hindered face (i, the convex fice). ‘Transition state B is destabilized by the electron-withdrawing effect of oxygen. Reference: Paquette, L. A; Youssef, A. A.; Wise, M. L, J. Am. Chem. So. 1967, 89, 5246, Solution to Chaper Problems + 89 “ loves ores ts 3 ; i h f PO OLR & jor atereomer 1% OH (e989) Solution: ms 14988N ores iawon te 8.03, NaBH on cis, O° can CHeele o5:18 (sr saserenmer sown) fae 5. mot poms i Steven oe on 8 bu ° 1% (swps3-6) Step3_ Ozonolysis followed by carbonyl reduction. Step 4 Selective protection of he 1 alcohol, Reference: Wovkulich, P. M.; Shankaran, K.; Kiegiel, J; Uskokovie, M.R. J Org. Chem. 1993, 58, 832. 3. Stercochemistry. Give tho structure and predict the stereochemistry of the ‘major product formed for each of the following reactions, Give an explanation for your choice of stereochemistry. 1. THOFPIg DET, 8. (MaO)CMes, aC — tests 4, a ne lo 2: HO=CHOH,MaCt 4a. 03, OH, 70°C THe 40. MoS, NaBH, 90 + Chapter $ Functional Group Trnsfomaton: The Chemisty of Carbon Carbon Bonds Solution: ante moatod Breast 1. 140HPry DET, t HBv0oH Cech wd Me 2, HxOnCHCHaMgc AF on Me, HO.Me oH " oH Me Me at cy 3. (MeO}gCMee, cau $2.05, EtOH, Tae 4, MegS, NaBH, Ho. 42 ohm Step 1 Sharpless asymmetic epoxidation. Step 2 _Anti-periplanar opening of the epoxide at the last substituted carbon, Reference: Evans, D. A; Bender, 8.3 Morris, J. J. Am. Chem. Soc. 1988, 110, 2506. Me “OL 8 THF SEH THE og ue BNGOH, W202 Solution: tacos more Me indeed 18 2. BHy THF Me Me B.NSOH tBu, o# 02" Me 8 ‘Soliton to Chapter Problems * 91 Stepa The predominant eyelohexene eonformer places () the Bu equatorial and (i the allyic-Me pseudoaxial to minimize Al strain. 5)n-Addltio of the H-B bond atthe leas, ater oxidation, to B. 1. TOs Ae (}0ET, tBe00H, Yom _ ti" HO 08S SheaaeA 2 Reo, THE, Solution: 1 THOHP He rom tettiawon yom naps GEC -20°C GO” as 0% 2. Red-Al (2 09) "08 (91 mixture of astereomers) THE, HO OH OH ¢ 05% [BPS group i cleaved during the Red.A' step. Reference: Nicolaou, K. C; Uenishi, D. J; Li, W. Si: Papahatjis, D. Ps ‘Chakraborty, D.K. J. Am. Chem. Soc. 1988, 110, 4672. il le CHEN, 15 °C SAK Sa ZorOK TBO Seton: 1” 0 M9 hie cone My 200K enn seme “Oy, Beg Be tment A sighted oh 0 an be D0 Hemet oy > tomy eT ati ioe * 5 AHO ° atic 1.2 sain esiabiizng 92 + Chapter 5 Functional Group Trnsfomatons: The Chis of Crbon-Caron Bands Reference: Collum, D. B.; MeDonald, Il J. H; Still, W.C. J. Am. Chem, Soe. 1980, 102, 2118. Select the appropriate reducing agent from thelist shown below. Explain your choice eyo reducing agent NaBH K-Selectie, or DBALH Bo Bnd Solution: — = a eT [Note that | 4-reduction is impeded by the adjacent disubsituted carbon center. ‘The sterically hindered reducing agent K-Selectride favors 1,2-addition from the face opposite the Roy substituent, Reduction with NaBH, or DIBAL-H fumished a product mixture in which the ‘equatorial allylic alcohol predominated. However, reduction using K-Sclectride produced the axial alcohol as the major produet (88% yield, 9.8 : I mixture of ‘diastereomers. Reference: Martin, 8. Zinke, P. W. J. Org. Chem. 1991, 56, 6600, 4. Reactivity. Explain the regioscletivity and stereochemistry observed in each of the following transformations. 1 Hg( Aes THE. Znaphe Ho Son sa Natt Cor Solution to Chapter 5 Pots + 93 1 root 9. . Hone Hg0¥ =n outa o iS . On ‘Bie. oe, . Mg ‘MgB Ce CB [em — . ‘0 PhCHeCHaMgBr pH Lewis acid-mediated epoxide rearrangement followed by ,2-addition. + fo AY we CL po 94+ Chapter $ Functional Group Transformation: The Caer of Crhon-Catbon Bonds ‘Solution: A a © ot eh if Q Hs "0 yeas ‘oH ° o ye io” HTH a Why is the Ppepoxide not formed on mCPBA epoxidation, although the B- epoxide is predicted based on OH-iected epoxidation? Propose a strategy to synthesize the B-epoxide, wel x ‘The unfavorable steric interactions of face peraci-hydoxyl group complex with the angular methyl group and with the 2B- and 6B-hydrogens results in the preferential epoxidation from the c-face Since an a-face approach to C(4) is favored, presumably attack of the sulfur lide to C(4)-carbonyl would yield the fface alkoxide leading to the P-epoxide afer protection ofthe hydroxy! group. Solutions to Chapter Problems * 95 12.05.CHice 1 a 2a. {UCH_SMeaL THF” 10% Het wel 2 so Hc 7 £0 Reference: Ekhato IVs; Silverton, J, V.; Robinson, C. H. J. Org. Chem. 1988, 53,2180. ma ono. WL 27) + moraa i oe ee ‘o> NaHcOn butter 0° ‘Ho BEEK t H q 07 warburgana Solution: ie Step 1 Selective epoxidation of the more electron-rich alkene (enol ether) ‘occurs selectively atthe acface to avoid unfavorable interactions with the f-face angular methyl group. ‘Step 2 Mild acid hydrolysis ofthe epoxy ether (a strained acctal) and the 1,3- dioxolane acetal Reference: Kende, A. $; Blacklock, TJ. Tetrahedron Lett. 1980, 21, 3119. 196 + Chapter $ Functional Grup Taslomtions: The Chemistry of Carbon-Caton x Bonds “f Provide @ mechanistic interpretation for the formation of the aldehydes shown below ur st “Toone on aoe mh we Solution Ho ju ey = 0 wii NAL ine Ho. we Jo. 3 Me Hd 48 YS ut ou Gu ho sire opin were “| 2 gor or 9 ve = 7 ol Xue Yo oe, Me = 0. 10 Lag %, - He ct sim) ~o¥ c 1 Me Me pig Ota ote DG" ‘Me Lon 0G’ iJ HO 5 be Bes fe é om a Reaction of the epoxy alcohol with LiBr forms. the halohydrin salt that equilibrates to the equatorial-Br conformer, which then undergoes ring contraction via an anti-parallel displacement of Br” (Step A). Proton transfer (Step B), although proceeding ata slower rate, leads to another anti-parallel Br isplacement (Step C), forming the minor product FI. Reference: Bergman, R; Magnusson, G. J. Org. Chem, 1986, 51, 212. Solaion to Chapter Problens + 97 5. Synthesis. Supply the reagents required to accomplish each of the following syntheses, Indicate the relative stereochemistry, where applica- ble, ofthe praducts obtained at each step. on HO: CHOH Solution: oH HO. /-OH Bilbo fol & NaOH HO 5 ort, Ho ov 3a Solution: 1 Lf 20g). NM) EBUOH ea), EHO, 2. mOPBA (1 ea) eannor CH, O°C workup 30, aqHoo3H Cx) NaHCO ‘workup er Step2 Selective epoxidation of the more electron rich double bond. Step a Acid-catalyzed epoxide cleavage. Step 3b. Formate hydrolysis. oH WORAAA eA 98 + Chapter $Fartional Group Taasfomations: The Chemistry of Carbon Cabon Bonds Solution: Niet Woe CO a pou Sou cone Tes Solan coon 109829 SE ne we SL. moan ons Oo Ncoaie See BHP iS made 5. MeMos, THE 6.4605, |4.P06. OMe ona SOLE We Tanager Tht -e ‘ate Steps 2-4. R-CH,OH to R-C(O)Me sequence. Reference: Machinaga, N. Kibayashi,C. Tetrahedron Lett. 1993, 34, 3738. Solution to Chap Problems * 99 a alerted fen ae eee rs : See, vo 7 * Breese | PL oue | 8 Ze | md Reference: McWilliams, J. C; Clardy, J. J-Am. Chem, Soe. 1994, 116, 8378, oe _ " on sie rane on Solution: 1a. tcBtp E40, THE So 2.MnOe Te MeOH, Naot on ono Hos oH On 70% ‘a, H_C=CHMgB (xs) The on BeaMHso, on 70% Ho" 100 + Chapters Punsionsl Group Transformations: The Chemistry of Catbor- Carbon x Bonds 2M M280 Wee on ‘OH 78% Step 3b Allylic rearangement via allylic benzylic cation. Reference: Holoboski, M.A. Koft,E. J. Org. Chem, 1992, 57, 965. e ° Doe, + Den we on ww to ores Sotuion: + (crop arte Co. BEES. Oe we! on AAS, C05 20° yy oa 2 faa. 3, TBSCI, mid, CHCle OL. 52.03, E10Ae, 78°C “4-1, Ny), Famylaloohot A “Bp PalOMg, 20°C Tr Me" ‘ores 0% (sepa 4) 9 2 0 On 0 6.TSOH, THF Me ores ePo Sores (elim 20) oF ° (Je Meo" Sores 0% (steps 5-6) Step S0-b Ozonolysis followed by reductive workup leads to the P-iketone. Reference: Wipf, P Lim, S. J. Am. Chem. Soc. 1998, 117, 358. *n chiral moty group, ‘Sng enantiomer Solution to Chapter 5 Problems + Hint: No SAE was employed inthe synthesis; however, bo eoquence dd Involve an alc alzopalntrmodiato 2.asymmetic 9 murat Pca ets betes Sree re " CHCl 3 PTS 6. TEC, py “Ga. BHy THE, THE DF. NaOH, MeOH sto homeinrm ¢ _B.NaBTe, DMSO sews ly SE oonaton Reference: For an analogous transformation, soo Altman, LJ Han, C. Ys; Bertolino, A: Handy, G.; Laungani, Ds; Muller, W. Schwartz, S.; Shanker, D.; de Wolf, W. H.; Yang, F, J. Am. Chem. Soc. 1978, 100, 3238. 102 + Chapter 5 Puntos Group Transformation: The Chemistry of Cabor- Carbon Bonds 6. Consider the reactions A-R. Assume that the Sharpless epoxidations proceed with complete «facial seletvity regardless of substrate, Select the best answer among the following choices regarding the stereochemical ‘outcome of each ofthe reactions. A 7 CO on He Ton Se > po Toe © ro CA01 eer Anum roel noe wot, asa Fre oer LA ogee AQA01 Foot oom oe Assuming tha the stereocentes, if any, in the starting allylic alcohols are racemic, which of the above reactions lead() toa mixture of enantiomers? (@ reaction A only (iv) reactions A and B reactions B and B, (9) reactions A, Band C (i) reactions D and F Solution: stereomers. Reaction D affords a tion proceeds with complete x= Reactions B, C, E and F lead to mixtures of single enantiomer since the Sharpless epox facial seloctvi (Only reaction A provides a mixture of enantiomers, Therefore, the correct choice is, 'b, Assuming that the starting allylic alcohols, if chiral, are enantiomerically pure, ‘which ofthe above reactions leads) 10a mixture of diastereomers? ( reactions Band F ——_(y) reaction B only (Gd reaction B only (6) all of them would except reactions A and D (li) reactions B and C Sotions to Chapter Problems» 103 Solution: Reaction A gives a mixture of enantiomers; reactions D, E and F give a single Reactions B and C lead to a mixture of diastereomers. Therefore, the correct choice is (i. “7, Retrosynthetle Analysis. For each of the following syntheses show (1) yout rettosynthetic analysis and (2) all reagents and reaction conditions Fequired to transform a commercially available starting material tothe target molecule Propose a synthesis of (*)-rans-3-hydroxypipecolic acid using the Sharpless asymmetric dihydroxylation procedure to establish the absolute ome on . : ‘00H (tans shycroxyppecolie aid Retrosynthetic analysis: PQ = protecting group -.0PG! gest POA AA opct > On “ u Paro A_cHo 104 + Chapter S Functional Group Transformations: The Chemisty of Caen Cabon x Bonds Synths “yee no wk, simran HOA, ews. = "OH "2 POG, NaQAc 0 Genzene, refx cio 40.0884, Chaat eo. coe OR. pu A N, as “ab. Rochelle’ OH a ee ox coupe-9) 5.060, (004, rOePrA Bo 09 fecOs rey oH a PucHOnee cisBate wo. Ton eat AA Aon FBLOH, HOT, OS Yoo once srs cyan ibe ato 7482600) Ps i 7.0, OMAP zg oto oho ove PCO ~ ALF Gray DH AOTE FBO ~ AJ ES 85% som “® BY ra toe 9. 000, CHO HO. oo pac Ge 10. MsCl EIGN, DMAP, MSO. “ Vib 80,0 ton OH oie t supe -19 12.8PSC, DME, nit Ta MONCL GPE Cece Co 18. CrOyH80, 16. 6NHC, (tans nyéronypipecoto ald 75% Solon to Chape Problems + 108 Step 3 Olefination using the stabilized Witig reagent affords the (E)-alkene with good selectivity. Step 6 —Acid-catalyzed trans-acetalization using benzaldehyde dimethylacetal favors formation of the six-membered ring acetal. Step 8 Sy2 displacement of the mesylate establishes the correct nitrogen stereochemistry. Step 11. Azide reduction to the-amine; hydrogenolysis of the benzyl ethers; intramolecular displacement of de mesylate to construct the piperidine ring. Step 12. Sclective protection ofthe 1° alcohol Step 15. ones oxidation ofthe 1° alcohol to the coresponding carboxylic acid, Step 16. Simultaneous removal of MOM cther and Boe protecting groups affords the T™, References: Kumar, P; Bods, M.S. J. Org. Chem. 2008, 70,360. b, Jourdant, A; Zhu, J. Tetrahedron Lett 2000, 47, 7033 Propose a synthesis ofthe following lactone using the Sharpless asymmetric ‘epoxidation procedure fo establish the absolute stereochemistry. eyes no Retrosynthetic analysis: mT? > 0 3 me, 2 oye = ‘eno— ‘bn0—' Ho TONQ 5 ~ 106 + Chapter 5 Functional Group Transformations: Te Chemistry of Carbor-Carbon Bonds Lreinaee Pe ron RS ron AERIS soya ve See 52% oF ranstr maclated 20% (206% STR ; ear ve Ton fe HP peel i — esa 3.NaH, BnBr, DMF TO OH 5. He, Lindlar cat. TO OH pana TOA OOH] BEES TO 0-0 wrcom |, NT | NT ae not iol Reference: Kang, JH. Siddiqui, M. A. Sigano, D. M. Krajewski, K.; Lewin, N. Es Pu, ¥.: Blumberg, P. M.; Lee, J; Marquez, V. E, Org. Let, 2004, 6, 2413. CHAPTER 6 Formation of Carbon-Carbon Single Bonds Via Enolate Anions Overview Chapter 6 focuses on carbon-carbon bond formations via reactions of enolate anions. Problem 1 stesses the use of appropriate bases for generating enolate anions. Problems 2-4 deal with selectivity issues encountered in enolate anion reactions, The syntheses of TMs in Problems 5 and 6 require the selection of specific reagents to achieve chemo-, steeo-, or enantioselective carbon-carbon ‘bond formations. Key Concepts 1,3-Dicarbony! compounds + Caisen condensations + Dieckmann condensations + Enolate formation + Baldwin's rules for ring closure Imine and hydrazone anions Enamines ‘Aldo! reactions ‘9 (B)-and (Z)-enolate stereochemical contol ‘+ Mannich reactions Enone synthesis + Michael additions + Robinson snnulations or ptr 6 Fomaton of Caon-Cnfon Single Bonds Via Enola Anions SOLUTIONS TO CHAPTER 6 PROBLEMS ‘The more challenging problems are identified by an asterisk (*). Reagents. Give the structures of the major product(s) expected after each step of the following reactions. Be sure to indicate product steroochomistty where applicable. 1a. LDA THF, -70°C 1h ACB on ee eonntirey rv Bes Aon ee oy eee % De enuyg | BHO Ae Cinema os rom BAH Step 1 Nitrile c-anion formation end alkylation, Step 2a Controlled reduction ofthe nile with DIBAL-H (1.1 9). Step 2b Acid workup gives the imine product which then undergoos hydrolysis to the coresponding aldehyde, 48.05, CHy Cie, MeOH, 0°C co 1b. Meas, MeOH . aq 10% Naz003, ae MeOH, reac Soltion to Chapter 6 Problems + 108 oF ° Solution: 18.05, CHG, oo MeOH, 0°C. 15: Me8, MeOH a ada Sondostton ° Reference: House, H.0.; Lee, JH. Cj VanDerveer, Ds Wissinger, J.B. J (Org. Chem, 1983, 48, 5285, 2\alute HORE werk — cy | Sam [Aa | Fagor te : ‘Step. Regioselective enolate formation, directed toward the sterically less encumbered e-postion. Reference: Riechers, 7; Krebs, C.H.; Wartchow, R; Habermehl, G. Eur. J (Org, Chem. 1998, 2641 tr 6 Formation of Caon-Cabon Single Bons Via Enola Anions 41. NaH 20.00), OME 2a, SiagBH, THF, 0°C Su. n 2b. HzO, NaOAG, HzO. D TE GOH, (149 (tea EeSiacworeoe 1 ty S02Ph tMeo,6 SP" 3 o Wieno yofsoon eer] teoee yao om a ets P90 cou ofsatone go 7% Reference: Nantz, M. H. Radisson, X.; Fuchs, P-L. Synth. Commun, 1987, 17, 5s CO Q ——1.CH;ON,ratux 48. aq HO, 100" BAe + AA, Beatie 2a. 00 NaOH (rea ea 2b. aq HO}, 50°C ome 3. mRPBA, CHCl, Solution: an ey i {EHON, rot oO Ny o 2. n0P@A Bray = ° 70% . x Solon to Chapter 6 Problems © 111 ‘08 = 2002 ° . 0 of Step 1 Enamine alkylation followed by iminium ion hydrolysis. Step2 Ester saponification followed by aciificat mn -CO;Na to -CO.H) and decarboxylation of the keto acids, Reference: Holland, J. M.; Lewis, Mi Nelson, A. J. Org. Chem. 2003, 68, 747. fa Op Msor,-16°° a Meck inne CL LY eco, Toe 880°C Nora Coat Gay 112 + Chapter 6 Formation of Cubor-Caron Single Bonds Via Enola Anions Solution: aso” 3.MsCl EN CHa, °C, pe Zoe | OHeOla.t — BPSOT TK Ft 495 (4 stops) Step2 Intramolecular aldol condensation; path a is favored (five-membered ting formation) over path b (seven-membered ring). Step 4 -Elimination of OMS. Reference: Falck, J. R; Manna, S; Chandrasekhar, 8; Alcaraz, L.; Mioskowski, C. Tetrahedron Let. 1994, 35,2013. AAAcuy tibia tluene ° CHO a LDA 1.08 6g), THF 78°C 2b: mabe 22. 4g HCI workup Solution: 1.8 Bu totone Ano mt" Ai 2a. LDA 5 ea) THe, 78"C s 20. nBuBe 22. 2g HCl workup Step 1 Imine formation ensures monoalkylation of the statin aldehyde 18. (imig),0=0, THF,0°6 1b, (MoO,CCH,CO,Ma, warm tt iene cont, Jeanichverig OO tSon an RBH Her a, 25. aq NalicOs wor Solutions to Chaper 6 Problems + 113 Sotion: 9 ta fice to e0-C0H.COs ce Sh aE ca 1. eo,CoH4c02hNe oo hod 1) nr rciwran ° on 2a, MoO gph cove ZENO Cran, TY come 00m Seanad oOo whe . oh Reference: Durham, T. B.; Miller, MJ, J. Org, Chem. 2003, 68,3. Hy POsMe 4.20% aq Hec-0 fo _MeAHHHEN doxane P 2 Mel (exc655), MeOH, 674 fH 3: oMr,80°C Solution 1.30% agHiC-0 MeO; 4 Ose Mente Longtime, sonar M® 9. ou. 20°0 °F ito excaes) “ a MeOH, fy oso tein cit. = Mel, = CO2.~NMe Step 1 Mannich reaction. ‘Step 3 The reaction proceeds via a nucleophilic decerbalkoxylation, Reference: Behare, E. S.: Miller, R. B. J. Chem. Soc. Chem. Commun. 1970, 402, 114 + Chapter 6 Formation of Cabon-Cabon Single Bonds cost .Na 260) NH 4.0304 cat), NalOg (excess) s8u0H, Eu "|, H0.doxare e 2 (PhaPAAAC cat) EQN, heat atone CH ga GO) 5b. E10H Solution: 4.030, (cat) * SO a.naezen.nym FO “Nat tercesy fH a BuH ERO Oh. NO. Sona 40 i 2 (PhaPishhca (cat) ort, (errand chy Cu sa (60,0 ” b a2. B0H f coset const 0.8 SEQ on Py ost Tear LVF ~ Ho cH 0 o no patiminaton an ‘posable Step 1 Birch reduction, Step 2 Wilkinson's catalyst (PhsP))RKCI allows the seletive hydrogenation of, the less substituted double bond of the 1,4-eyclohexadiene system. Step 5 Under these thermodynamic conditions, equilibration among the enolate anions leads toring closure to form the thermodynamically more stable congue yen e © 6 ‘gl, Eun THs oye | wee ‘ ee 2, TESCI, imid, oo CHeCl, rt 3. BH, MeOH, THE Solutions to Chapter 6 Problems + 118 Suton 5 aide. roy ine tt? @ \, er nan en Y T830 2. TES, imidazole GHC, TUB, MeOH, THE Reference: For an analogous synthesis, see Gaul, C Njardarson, J. Ts Danishefsky, S.J. J. Am. Chem, Soe. 2003, 125, 6042. Selectivity. Show the product(s) obtained or appropriate reagent(s) 10 be used foreach step ofthe following transformations. 1 1140 HE oe EhaetE No iaog A ‘Ti wopeh. Oren a, coke FAX A piperidine, toluene, heat Solution: on emg ni a CS te tat > Oe a aw a 75% coupe 9 21a (20a Pioetca) Hoke iperiine folvene host 116 + Chapter 6Foomation of Caon-Carbon Single Bons Vin aol Anions Step3 Hydrogenation of the terminal double bond Step 4 Intramolecular aldol condensation favors the five-membered ring over the seven-membered ring (see Baldwin's rules for ring closure). Reference: Mehta, G. Pure & Appl. Chem. 1990, 62, 1263, SEN 0 zeae By ke Sotuton: eg tivcyeammo, gM we CHigeN CHSC, HO. wT? 2. KOH, MeOH 7 eat iPr cox o ser Pr 4 oO Apo [Howe] Me" ° 6 8 36% Step4 Intramolecular aldol condensation favors the five-membered ring over the six-membered rng Reference: Mehta, G.; Krishnamurthy, N:; Karra, 8. R. J. Am. Chem Soc. 1991, 113, 3165, * ° on Qo om 70% overall Soltis wo Chapters Problems © 117 Solution: ° nm oo 1 ethyone ye TsOH 2a LDA THF, a0 2b. TMS-propargy| bromigo on Ne TMs 8. KOH, MeOH (Gesiyation) ‘4a.LDA,THEDA THE, -78°C = mon no\=-c0,0H5 4c. HE Hg0 workup fant 70% overall Step 4a Nofe the use of LDA instead of ether -BuLi or MeLi to form the lithium acetyl. Step 4e Protonation ofthe lithium carboxylate and acetal hydrolysis Reference: Trost, BM; Shuey, C. D; DiNinno, F., Jes MeBlvain, 8.8. J. Am (Chem. Soc. 1979, 101, 1284. A A ° 1 ott fo2500 1053, ee ee 7 . ae Te loate ue Zeno” By Won ice Z Ru eR word « Cxth cEaR CUM Mee CO2Ne toluene Do stop tf a pl MeO oe a weg" 2 mopMe io Sion ‘ on Tt Toate Toe : ester hysrlysis, 118 + Chapter 6 Forztion of Cabon:Caton Single Bonds Vis Eola Anions Step 1 Michac! addition Step Dimethyl acetal hydrolysis; ester hydrolysis followed by decarboxylation; intramolecular acid-catalyzed aldol condensation followed by dehydration Reference: Danishefsky, S.; Zamboni, R; Kahn, M.; Etheredge, S.J. J. Am (Chem. Soc. 1980, 102, 2097. fa. uns, Ti 70° ows jeune Ayo fisttinne ei BHOnNWOH Signe OS, Stone” B, Solio: HOTS se.uncrwsjp OTBS 4 coves: Beye ie A ons 2108 08 ° wea eal se ¥ ores 2. L604 fo A: Nar, oa ee Te A @ steps) 0% Step 1 Regiosclective enolzation due to steric shielding ofthe a-OTBS group followed by O-silylation ‘Step 2 Bromination of the silyl enol ether to give the a-bromo ketone. ‘Step 3 Dehydrohalogenation Step 4 Conjugate addition of HOO” from the less shielded «-face of the enone. ‘Reference: Hartung, R; Paguette,L. A. J. Org. Chem. 2008, 70, 1597 Soon to Chapter 6 Problems © 19 3. Stereochemistry. Give the structure and prodict the stereochemistry of the major product formed in each stp foreach ofthe following reactions. Give ‘an explanation for your choice. dys -.Chxg80, MosNet one 70° B08 a BCHCHCHO done Sert0=25°C agate) 6: NaOH workup Solution: .cHycl.cHo ohyA nese sock FE pro SOMEONE 5,04 ee CeO, 78°C Some PDE NOH ° AP a nt bende cre Mo 5 moe e oon a a 2% 06 9555) _Siappronch tavored Stepa E-(0)-boron enolate conditions; reaction with RCHO results in a 2,3- ‘anti aldol produc. Steph x-Facial selectivity may be determined by the stabilizing effect ofthe H- bond between the benzoate oxygen with the aldehyde hydrogen (see Tetrahedron Let. 997, 38,33). Refere Paterson, 1; Wallace, D. 1; Cowden, C.J. Synthesis 1998, 639. 12, UcH:COMe THe, 78°C 1b. HO workup ZMesOH som enone . Ceo ‘ta, LICH,CO,Me " ° ate Mesos ac" soe? ho : sor ber Step? Fomaton fhe c-fised 56g hemodynamically pei. The fcc contin eal foe eqbraen pment of on of be Bilge poston va enol tated de be sey a the ytogonsoated ye enn nd eer sea Reference: Thompson, S.K.; Heathoock, C.H. J. Org. Chem, 1992, 57,5979, st) aaa 5 ar ve ‘acetone, THF, ~40 °C oR a 1a.LDA TMso. SMe eee on = = Ee ioe iPr Solutions to Chapter 6 Problems © 121 MS Pon reuros | SS ALY areal td see “stained homiacetal (rotieoates) (not saated) a Step2a_ Regio- and stereoselective epoxidation of the mote nucleophilic silyl ‘enol ether (double bond). Although it seems at first counterintuitive, the ‘epoxidation occurs from the fface (build a molecular mode). Step 2b Silyl ether hydrolysis followed by hemiacetal equilibration leads to the Keto-aleohol. Step 2 is a modified Rubottom reaction (Org. Synth, 1985, 64,118). "Note: The acetal functional group survives the reaction conditions in Steps 12. Reference: Shi, B.; Hawryluk, N. As Snider, B. B. J. Org. Chem. 2003, 68, 1030, gio 1.7601 ying oF xore, BOF BN gio 1.Te0\ prtine BKOrBu, FBuOH Pe S0 F otoe 80% Solution: ono gure 1.01 _2.KOFBu 40H pie a 10) Pt, - CRA, -ors 4 D1 om 122 + Chapter 6 Formation of Carbon-Carbon Single Bonds Vi Eclat Anions Enolate formation at the less hindered c-position followed by intramolecular tosylate displacement affords DI oir @ poo oom” Ak, gr GHeoTs TsOH,C. ; . vr oot Of the two chair conformations for the cis-keto tosylate, 1 and 2, only conformation 2 where the angular TsOCII,~ group is axial to the ring bearing the (C=O group can undergo iiramolecular displacement. The rate of a-enolate vs «-enolate Formation plays a lesser roe in determining the regiochemical outcome of the reaction, Reference: Mukharji,P.C.; Ganguly, A.N. Tetrahedron 1969, 25,5267, 2 REARS 18. 23tdromapepene oy tscrenenopne __g Cc) BLAM THE. -79"0OS ey pt ie 25. NaOH, 0 warp Solon to Chaper 6 Problems © 123 Sottion: e¢ yy EW seam as . = WAL ody ML Me 2 rie aunty Hi raeworo ssprae dase by eae on ae eres se ® Me © cove Sca Reference: Evans, D. A. Bender, S.L.; Mortis, J. J Am. Chem. Soc. 1988, 110, 2506, *t a. NAOH, HzO consent, A ‘ssa te50, wr, 80s HO(OMe, MeOH FA (M,>95% 00 F2 (5), >95% 00 124 + Chapter 6 Formation of Cabon-Caron Slagle Bonds Via Enola Anions Sato: so cone soa Con Add Mos, MeO,C_~(@.0Me ° ‘ ie wo oooh Mog a RS | a2 mote FE(OMs i ° Que oor | yooA 4 on arafor pee {freon {acon sa onsetarenion Met wWo.e_~3.e ig voy A [done] two MOP Bas-aaerenion Saponitication followed by alkylation with dimethylsulfate; no inversion of the chiral center. Acid-catalyzed reaction (F2) ‘Trimethylorthoformate forms a stabilized dioxocarbenium ion which activates the Iactone towards attack by methanol with S32 inversion at the methyL-substituted carbon, Reference: King, 8. A. J. Org. Chem, 1994, 39, 2253 4. Reactivity, Propose mechanisms for each of the following transformations. Q ost Q NA snc nf Ot avene Me min Me 22% Solutions to Chapter 6 Problems + 128 a FOEt roe a ee a dN Aow - A a cl ot ae Mer pet Mer Be % Reference: Padwa, A; Lee, H. 1; Rashatasakhion, Ps Rose, M. J. Org. Chem. 2004, 69, 8209, b. 00, 2. E10Ne EtOH Tro ‘o ‘coret werkup boxe Solution: aH oy , a EtONe DL coset * EON J Ue ] oer op “oon 07 oe ‘Diecimann condensin ‘o coset ‘The driving force for the (elimination of the ester enolate is relief of the cyclobutane rng strain. 126 + Chapter 6 Fomaton of Cubon-Caton Single Bonds Via Enola Anions ° I agHl cH07F07 THE, AOE Reference: Boeckman, R. K. Jr; Bruza, K.J. Tetrahedron Lett, 1977, 4187, “ ° OF oa REP. Bhs CL + Aeon SE omve ‘coset 7 a bow = CTY) sae asa Solution: ° pen eq & oO er, T + ALcoe Hee i ro 0 copet ‘on ‘cope: moeaton ‘coe Reference: Gamrill, R.B. J. Org. Chem, 1979, 44,3988, Me ores 8 Tey Cite cHHOMel2 Solution: mo sitet ores 4 i ao Meo. A crower =meoricl com benzylic oxecarbenium ion ° TBS, H Meo, "one 128 + Chapter 6 Fomation of Cabon-Carton Single Bonds Vis EnlateAnions ‘The electron-rich aryl ring assists the Lewis acid-induced formation of the Feaetive intermediate oxocarbenium ion. Ring closure oceurs via Mukaiyama- 'ype trapping ofthe oxocarbenium ion by the silyl dienol ether, Reference: Morihira, K.; Seto, M.; Furukawa, Ts Horiguchi, Y; Kuwajima, 1, Tetrahedron Lent. 1993, 34,345. * ° CO 4. Mal acetone, yang eta yo Puce 80 the Resist Setuion: ° ° ° ° QO 1. Mel a 2.NaOH Cy te rt yo semen Ne N MeN” NHB Me Md Me Me Me 1 Ps = waoge ier ME say, NHB Step2 Hofmann climination/Michael addition equilibrations. Acrylic acid functions as an efficient and selective trap for dimethyl amine vs. ter- butylamine, allowing for facile control of the equilibrium between 1 and 2 Reference: Amato, J. S.: Chung, J. Y. Ls Cvetovich, R. Js Gong, X McLaughlin, M.; Reamer, R.A. J- Org. Chem. 2008, 7, 1930. Solon to Chapter Problems * 129 5. Synthesis. Supply the reagents required to accomplish each of the following syntheses, Give the structures ofthe intermediates obtained after cach step and show their relative stereochemistry where applicable, cout Post oO — ° Gsm Ge ; . : OCs + CR 4 conet "se" Cogs TE ‘3. KOH, HO(CHz)20H Ho 8% tee 86% 4. ETOH, HS (at) Step 5a Dieckmann condensation Reference: MeDermott, T, $; Mortlock, A. A; Heathoock, C. H. J. Org. Chem. 1996, 61, 700. On=- of ‘Solution: Onis CS = = COs ' CO ss. 4-H SHO 20H Hoo Soo -10°¢ a G0,et on — 3yT ot os Solution: Gone 5 Tsci COxEt 28,03, MeOH Eun CHCl -70°C 7 BME ays NBR 22% 3a, Na, EIOH, NH) 96. 17, H:0 workup 73% Step 3a Chemoselective reduction of the— CO:Et group, Step 3b, Selective latonization to the six-membered ring lactone, Solon to Chapt 6 Problems * 131 Reference: Clark, R.D.; Heathoock, C. H. J. Org. Chem. 1976, 41, 1396, 10. KsCOp, n-BugNBr toluene Then ° 3.KOH,H.0, heat “HO(CH)OH, TSOH Ey ° 6a. LSolectide, THF “Go.NAOH, HeOe ? THF HO nor £ on 2 ‘coset _ Hy ‘coe Solution: 1a. €10},0=0 i 0 Nae, EIOH 0a NCX_COnET ‘coset Tb, Chal excess) (CH EN, Feat Chis ‘cope ‘ooze ‘Kroovenagel canntaton 132 + Chapter 6 Fortin of Catbon-Caron Single Bonds Vin Enola Anions b-% Sotuion: st 0, cr voor a6) “Eg sow se wet xo ee oe on terminy ptt maaan joie, Siorieneerincae’ SS abo — ado) at sont en eS ‘The tad Cabo condensation eneires thst ony mone ake he acne eben ru oor 410A {yasen, Biohnea VSL 9.7001 = Treobutene = “C0 rer Cow J Solution: 1. Mats, E40 2. nO (excess) hexane ‘CHO Solutions to Chaper 6 robloms + 133 om oon wT te mn en on sno,cousc0.e 2a. oH. Heo a Ri, BocoHCOse jartoaoe aA NaQEt, E1OH 2b. HCI, heat haat 10,0 coe (C02) Cot os gen oa. 800%, mt ha oAny cuts reatttent Reference: For analogous synthesis, see Helal, C. J; Kang, Z; Lucas, J. Cx Bohall, B.R. Org. Let. 2004, 6, 1853, cue, Ny r yoteoome Py Soto: ty rappaeieg OS 24.04 MeOH, 70° y ae Baits come HR Re cote BMS He POM Tay eromige™ HCN P COM sone One fom, veo relies on ce, ob, 1 ro Hg rac eye OR0 | oe eA, Zien NBN ”" 0% rom NaBsoN) 134 + Chapter 6 Formation of Cabon-Cabon Single Bonds Vie Enola Anions Hey, uae 5H (2008) ise AE py ee Paton re at) Oe Tie MeOH Step3 Reductive amination of the aldehyde formed in Step 2 followed by intramolecular transamidation. Step4 Amide and carbamate reduction Reference: Fleck, T. J; MeWhorter, W. W., Jes DeKam, R.N.; Pearlman, B.A, J. Org. Chem. 2003, 68, 9612. 6, Retrosynthetic Analysis. Outline a synthetic scheme for preparing each of the following target molecules. Show (i) your retosynthetic analysis, and (ii) all reagents and reaction conditions required to transform a commercially available starting material into the target molecule. ot, we, Retrosynthtic analysis: cry © cH Oe “ome, ¥ yh ss O.. 0% é€ che ° 0 Synthesis: 19. LOA, THF, 78: c 1. alytbromie| om ‘chy 2 080s (al), NaIO, (oxoss5) doxano, HO ¥ ° ‘o 5. roa 0, cy ee ore u Referens: Foransogos satis, se lve, Dt. Wang J J Org: Ch Doonan Ts » ‘ aN, Aerasymtac anays iM Pn ¥ ah cH FO a > tame + Cho > ( + ko ¥ cH ° tot Topnene on: Sates: Kopees on, sm, one cont tot Reference: Robinson, R. J. Chem. Soc. 1917, 762 CHAPTER7 Formation of Carbon-Carbon Bonds Via Organometallic Reagents Overview Chapter 7 deals with the carbon-carbon bond formations via organometallic reagents Problem 1 emphasizes the utilization of various organometallic reagents for carbon-carbon bond formations. Problems 2-4 explore the selectivity of reactions involving organometallic reagents. The syntheses of TMs in Problems 5 and 6 requite the selection of pecifie organometallic reagents to achieve chemo-, steeo-, or enantioselective carbon-carbon bond formations. Key Concepts + Organotthium reagents + Organomagnesium (Grignard) reagents + Orgenocopper (cuprate) reagents ‘© Conjugate (1,4) addition with cuprates + Preparation of enones + Organozine reagents ‘9 Reformatsky reaction © Simmoas-Smith reaction + Organoboron reagents ‘© Carbonylations © Matteson's boranoe ester homologation © Brown's asymmetric crotylboration + Pallagium-catalyzed coupling reactions ‘9 Heck reaction ‘Negishi reaction Suzuki reaction Sille reaction ‘Trost-Teuji reaction Sonogashira reaction 16 Solutions to Chaper7 Problems * 137 SOLUTIONS TO CHAPTER 7 PROBLEMS. ‘The more challenging problems are identified by an asterisk ( 1, Reagents. Give the structures ofthe intermediates obtained in each step and the final major product expected for each of the following reaction sequences, Be sure to indicate product stereochemistry where applicable. 4. MagA 012100. (eat) ochcHci Gey CCH OO Cate CON be Solution: a. Mol ropa (eat), OH lp MOMs H nets con —CeeCpe at, ah ‘eIcHCH-CI ‘aes ud co Reference: Negishi, Ei; Bagheri, V.s Chatterjee, S.; Luo, FT. Miller, J. Ri Stoll, A:T. Terrakedron Lett. 1983, 24, 5181 ° anaes ors Hace, 2s ‘Solution: homiacetal 4a. MezCull E10, THF, 90°C CS °. ©. °: db. ag ucla 40 3 aiNiucivetip —__ c “TS pia (.0en) oases 2 Rootes sonore “te 8 Em Step 1a Me,CuLi adds in 14-manner. Step 2a Contolled reduction of the lactone with 1.0 eq, DIBAL-H provides the Iactol B. Referen Paquette, L.A; Chang, J; Liu,Z. J. Org. Chem. 2004, 69, 6441 138 + Chapter 7 Formation of Cabon-Caboo Bonds Via Organon Reagents 1 (Moo)MeNHHo}, OCC EQN, CHeCip 2 ay OA cont [ce Al 7 ‘THF, 78°C rs be semua 4] CX co “oid ne Eo ¥ Me egSi © table ntermodiate 49% (stops 1-2) ‘Step2._ The alkyllithium reagent add only once to the Weinreb amie, Reference: Piscopio, A.D; Minowa, N.j Chkrabory, T. K.; Koide, K. Beetinato, P Nicolaou, K. C, J. Chem. Soc., Chem. Commun. 1993, 617. a, Pd (OAce (cat), CO, Py EGN,OMF, MeOH 785 Solaion to Chapee 7 Problems * 138 PPh ProP | OT! Pe a (OAc)e (ct, CO, PPh EQN, DMF, MeOH _Sseype casoyiton GO2Me clnination = PnP)-PaO) no" ‘B0—! 2 7% Reference: Nagamitsu, T.; Sunazuka, T; Obata, R; Tomoda, HL; Tanaka, Harigaya, ¥.; Omura, $ Smith, Il, A. B. J. Org. Chem, 1995, 60, 8126, 12. KN(TMS)a, THF, 05 4b. ThNPh « 2 MesSrSnileg THE oy Pane tet ows a peo 140 + Chapter 7 Formation of Caton-Caron Bonds Via Orgone Reagents Solu COL amerign CE Atos re G MO. ¢ Patri a e aman bres tras om ° ° cS sree ss tres Tras om 5 Reference: Su, Z.; Paquette, L. A. J. Org. Chem. 1998, 60, 764, t ‘a. (sopenty2nCl (21 2a) PPa(PPhg) eat), THE, Pe ear) 5 ‘T8s0" 16: H-0 workup 2." CiO,,Hs80x, #20, acetone 5 ‘Solution: ‘8. (sopenty2nCt (24) Pappa Ten oy nb 880 Te RO wep 0 2.605 14804 ws Te 0 50% Reference: Rodeschini, V.; Boitesu, J-G.; Van de Weghe, P.; Tarmus, C: Eustache, 1. J. Org. Chem. 2004, 69, 357 Solulons wo Ciaper7 Problems + 141 0, Meo UH "BH, THE 2. Pha, PoP ha (eat) o 2MINaZCOs Sn gy NEOs ge DME, rx Solution: Meo °, Meo LH cx ‘a THE Oo ° 2. Phe, Po{PPha)e (eat) MEO 2M NagoOs DME, refx a Slep 1 Regioscletive hydroboration ofthe terminal alkyne carbon. Step? Suzuki coupling. h 2 eBuLi (2109) THE, 78°C Me DNF er " nod eHow 142 + Chapter 7 Formation of Carbon Carton ods Via Organomet Reagents Solution: We a sou (tea) Ye Hs fo, SHU ufos or ey pow] > neh sate fo-owe ez Mo 9 Yo ou sou ma) Hn | eee " Hae o yore epe-a0re i 42.104 @en)-TH.-70-C 190-6 16: (GenS Cantonal) Any Eines ere ry 2a. LAM, NaOWe, THF & BLAH Nsome. ro Solution: Any sume any 10.(CHOh Cie o0ST | Te He HO woes 23 LA NOM Ana en Bile Y oe 1 aah Reference: Zhang, Y.; Wu, G.; Agnel, G; Negishi, E. J. Am. Chem. Soc. 1990, 112, 8590. a. £Butgol 2.1 69) JA este inte ‘Deataag nici ° Solutions © Chapter? Pablems + 143 Solution: ° 1. tBuMgC! 2.1 ea) dna “ete ie er | rr 4 Stop a Cul and /BuMgCl generate an in situ organocopper reagent which adds only once tothe acy chloride functionalities. Stepb Workup protocol ; a ag armen es “Sac , ae nom a - pearaatee eS Se ° coo ‘ Reference: For an analogous synthesis, see Alibés, R.; Ballbé, M.; Busqué, F; de March, P. Elias, L; Figueredo, M. Font, J. Org. Lett 2004, 6, 1813. Me .958N ‘const S89N yy OO =e arco SN Tirooee PAPPRA aL) IGPOL Tie ioxane, 05 Solution: Me 2.908N 0 wesc Tet [orcs wi 144 + Chapter 7 Formation of Carbn-Carbon Bonds Via Orzanometlic Reagents Me 1 Yoowet MWe Me PaPhpe eat” eso“, KPO co.et dioxane, 85°C Le Ti Reference: Taillier, Cs Gille, Bs Bellosta, V.; Cossy, 1. J. Org. Chem, 2008, 70,2087. ALNaSoPh, LOH ‘2a mCPBA (1 eq), ORD, °C 2b. arm ors ‘ser 2, MOPBA (tea) Chl, oc £ Pn, 9 EtOH co 20. warn ° °’ =PnseOH synthesis, see Clark, R. D.; Heatheock, CH. J. Org Chem. 1976, 41,1396, Pa hyo apr hae eat THF.A0°C oly PPP) eat) THE, 90°C Troe: Tracton ° Solaons to Chaper 7 Pas + 148 A Pete AL Drone + Peo GgoMe 4 * co, "Nt ally: Pd complex “M&O” 75 MeOH No NT Reference: Tsuji J. Tetrahedron 1986, 42,4361. cts Hao. ‘on _(PhyPaPa eat) (00 (atm) 78% 00s, DMF, 80°C Solu 90H 0H 003, BMF, 50°C “eo (PhoPPdCia(cat) M0 : a 00 (4am) ° ° 7% Reference: Covell, A; Stille, K. J. Am. Chem. 1980, 102, 4193, 12, LHMDS, THF, -78 oma 1b. THN > 2. He, PIO (a0), FBUOH gre F 50.0% — ee ae 90% eneroee, 94, feat Q ; Parker, M. Hl Armistead, D. M3 Shankaran, K. J. Org. Chem: 1994, 59,332. 2. Selectivity. Give the structures of the intermediates obtained after each step, and show the major produets obtained for each of the following transformations. MeTi(OHP Hs, | ° 0 ‘Solutions oy MoT4OHP 5 BO we g ° "0 A . ors EOF anonuannore eB rd 6. NHYCL 20 me Fi= PMBOICHSs Solution: on oe 2. Red-Al (1 6, THF, 0 Mo RECA coro 8 R= PMBOICHy Sotions to Chaper7 rolens + 147 bb-78°0 eNHYCL HO, 70% Reference: Hoye, T. R; Humpal, P. Ey Jiménez, J 1; Mayer, M. J; Tan, Ls Z. Tetrahedron Lett, 1994, 35,7517 8. ThaBt, THE (eh alaion) b.KGN, ‘ ©.(CFCOK0 pom, NaOH, HO ‘Solution: 8. a8, THE ah atiny b KON, (OF 600 | ~ 6. NaOH, HeOe CI a 20% Reference: Peller, A; Smith, Ks Hutchings, M. G.; Rowe, K. J. Chem. Soc., Perkin Trans. 11915, 129. 148 + Chapter 7 Formntion of Caon-Caron Bonds Vis Organmetic Reagents 3. Stereochemistry. Predict the stereochemistry of the major product formed foreach ofthe following reactions. Give an explanation for your choice. HO aoe ‘OPM. MgleEO ose 20:1) Solution: HO ae “Sopa Ngee opus. sn 0:1) Reference: Nagamitsu, T; Takano, D.; Fukuda, T; Otoguro, K.; Kuwajima, Ls Harigaya, Y.; Omura, $. Org. Lett. 2004, 6, 1865, . & Aer, Zn (10q) 70% ol =e An Set a] oe CICHACHACI ‘Aiea | Zrce (Tea) Zack ( eal Ans. 8 10% Step Negishi reaction; transmetallation (Al to Zn) followed by cross-coupling of the resultant organozine species with the vinyl halide. The Pd(0) catalyst was generated by treatment of (PhyP)sPdCh, with DIBAL-H, Reference: Negishi, E Okukado, N:: King, A. 0. Van Hom, D. E; Spiegel, BLL. J.Am. Chem. Soe, 1978, 100, 255. Solution to Chapter 7 Problems * 18 bo c 1 Sn 100% Pap 20 pic onsen. THE Bonus bho 3 2, } Coco } 3 4 Me Me Ax Me Pa Phals (cat) Me THOT eters 89° ; ° wom ‘Steps 1-2 Corey-Fuchs reaction: aldehyde to terminal alkyne transformation Step4 Suzuki coupling + Vyryan, J. Ra Peterson, E, A. Stephan, M. L. Tetrahedron Let. Referen 1999, 40, 4987 ‘ if anon ro60, CL Ma MOML Egy 820) oa woes) 9 recom wee Sh, — mee 20 se. ag NHC workup 150 + Chapter 7 Formation of Caror-Carbon Bonds Vie Organometlic Reagents Solution We eo »nc-come TBs0. 1. MezCuls mL HMPA Oh ee Saati 0 HO ‘ou cou] up Mayo, 2TH recom “tea (1200) recom aoe J=MeoH chee | © is ot nm Es A © 6: °: a Oo ‘ O ease) om Step? SiyLeterkydolysn and lstonization, Mo 0 teat and Yo Oe epinetzsbon rane —" e ; a we 3 I, e So oe) (equal ovr) = fees ter confeatonal aun me Reference: Galano, JM. Gérard, A.; Honoré, M. Tetrahedron 2000, 56, 7477, Sotions to Chapter Problems * 151 SL Mecutsncoe, zn, THF 2. pTeOH (ea) CICHeCHeC ‘a. Fadl foxcose), THF Tine 2 Roches sak work 44 (PhgP)gAM, Ho, PBUOK, THE Solution: 1 MeCHEICORE +209 ay 8 i E1O,0. Me 2 [ue A) B10: 9H a co a 2. pTsOH (eat) TReteman ICH CHC conaneaton n OMe ome ee OMe on Me 3a. Nal, AOR),), THE (Reo) 4. (PhgP) ANGI, He, BUOK, THE Bb Rachels salt wworkip Owe ion Me Hoos ‘oh P| te Owe Step2 Dehydration of a 3*-benzylic aleoho. Step 4 Conformational bias minimizes interactions with the indicated Ar-H and sels the stage for diastereofacial differentiation in the directed hhydrogenation of the double bond. Bese-promoted attachment of the alkoxide to thodium gives the product with high diastereosetctvity. Reference: McCombie, S. W.; Ontiz, Cs Cox, Bs Ganguly, A. K. Synled 1998, sal Se eee ore one see enemas “tapi bc sen — re OD Ae Ble F Teg ° ° oa 2. ane a S ° 2, x Soeso’ | Gru] 7 ane ° w Te" co a Ss ee kG | -holaton east the enprosuct clan ans tothe ro ce _ eX _ 29 oan iy - foe ° on & ‘Tho Folin Anh model ree “Smtoenclocey A rationalization of chelation control in carbonyl addition reactions is given in ‘ec, Chem. Res, 1993, 26,462. Note: The above interpretation of the Felkin-Anh model differs from that proposed by the authors in that the methylene group of the starting Aldehyde is sterically more demanding than the C2) ether oxygen. Solution to Chapler 7 Prabams + 153 Reference: Mukaiyama, T. Suzuki, K.; Yamada, T.; Tabusa, F. Teivahedron 1990, 46, 26. “a. Supply the missing reagents and structures. AA Sa. Mpe).8 oH, TMSO. Oh 2 steps EL0, -78 °C a eta eo 81, WENGOH HDs &, poe uso. ou: M20" a) Sotution: 18. 980N, THE Oh 1. NaOH, HO “ODL, ee Seton aac) st a (enustoril lohy) aa Mpo)8 cH, so Votecses E1,0,-78°C Ct #Bu00H Be WaOH Fee Meo" Cele WSO, Reference: Kouklovsky, C; Ley, S. Vs Marsden, 8. P. Tetrahedron Lett, 1994, 35, 2091 184 + Chapter7 Fomation of Caton-Caton Bonds Via Organometallic Reagents 4. Reactivity. Propose a mechanism foreach of the following transformations to explain the observed regioselectvity and stereochemisty, A ee SiMe, __ACy 0 { ae Me a —— - - te |e, SL, Rael icant) en i ace sons Oe at ° ° by é 7 | CY mar Q — Qe ad Ssiey “MO | ud ‘sioy St ho ene meet EES P acid-induced Pe 6 == ¢ e ‘The acylium ion is repioselectively attacked by the double bond to give the 2° catbenium ion, whieh rearranges tothe more stable P-silylearbenium ion, Reference: Mikami, H.; Kishi, Ns Nakai, T. Tetrahedron Let, 1983, 24,795, Solutions to Chapter 7roblams + 188 . CO + com anime OY, . MeOze A LP haP iPad (cat) aon 1 oxddative addttion 7coMe tae \ ve _m \ MeOzC = Reference: Zhang, Y.; Negishi,E. J. Am. Chem. Soe, 1989, 111, 3454, Me Mo ore BUF b= ‘DMF, HMPA o ot 0% ve “up On wer noun” — a HER we ‘seg ce 1 dadation 156 + Chapter 7Fomiton of Cafon-Carbon Bonds Via Orpaometailic Reagents ‘The ally silicon bond is cleaved by Muoride ion under anhydrous conditions to produce an allylic ‘nucleophile which atacks the enone ina 1,4-manner. Reference: Msjetich, G.; Desmond, Jc, R. W. Soria, J.J. J. Org. Chem. 1986, Sirs a sn, E80 d a oom Soation: 4) 2.01209, sam, EMCLBOD, | — d wore ASR a, | Ta ren “¢ LA 9 °o fe aes * \ 05% Nocleophilic attack in a 1,6 fashion by the allyslane double bond atthe doubly activated Michael acceptor produces A containing. the silicon-sabilized cearbocation. Loss of the trimethylsilyl group generates the exocyctie methylene moiety Reference: Majetich, G; Song, 1-S.; Leigh, A.J; Condon, 8. M. J Org. Chem. 1993, 58,1030. Solaion to Chaper7 Problems + 157 5. Synthesis. Supply the reagents required to accomplish each of the following syntheses, Show the structures ofthe intermediates obtained after cach stop and their relative stereochemistry where applicable, Pt Oe one f ‘Fa. Mei Et Teneo te. HE 0 “workup) 2a. UA, 3a, MogCuti (excess) Naot rt ae" aoe on 282 say, ale Be.NHGL HO i (workup) ‘Step 3a A large excess of euprate reagent is required for this transformation. Or O8 ‘Solution: Ow Sie [O-=] Ae ea [Dm] smee OP at OR, Step Ib Alternatively, Cul can be used fo generate an organocopper reagent ‘which would work equally well for & controlled 1,2-adition to an acyl chloride. on pre He Wo A on 158 + Chapter7 Fomaton of Carbon-Cabon Hons Vi Organon Reagents Solution: a SU EDA own PISO ayo pny THES Talla Oe wtp ascoue unt On (workup) oo Retrences: (9 Takao, Sahar; Ognsavar K, Sy 190,488 (b) Takano, 8; Murakami, T.; Samizu, K.; Ogasaw Heterocycles 1994, 39,67 BA nett OS rs ores Solution: Wom BRAN nt PAPAIN LAHCH HOA res RN 7c Sonagastiareacton orm OBS Reference: Nicolaou, K. C Webber, S. B,J. Chem. Soe., Chem. Commun, 1986, 1816. co’ — oot ‘consider he Solution to Chaper 7 Problems + 159 eH (oky~ 17). od stong bese D> nots + coy Ee mavceeye fre nas ce He samc uel Reference: Imamoto, 7; Sugiura, ¥; Takiyama, N. Tetrahedron Lett, 1984, 25, 4233, £ Messi MOS yg oh Solution: M8 Me amoreaiteg — MS me CHCl, 0° B.NaOH work O 646 Mossi 2 MezCutctgtis excess) “9 te THE, 75" 060°C aq NOH, NHGr yon or Epoxidation of the more electron rich trisubstituted double bond with mCPBA proceeds stereoselectively. The bulky Me:Si group blocks the a-face of the trisubstituted double bond Reference: Hou, J. Ri; Wetzel, J.M, J. Org, Chem, 1992, 57,922, 160 + Chapter 7 Fomation of Cabor-Carbon Boads Via OrganometilieRengets ® e oe wo Aon emo RT & Solution 1a. DSO, (COC) Clo, 78°C 4a, CpezHiCne 1b. EA, warm 100°C (ea) i ie. aqNaliCO,, THO A He THRO A_OH 2 BhaP, CBry Oe Sa Be 8a, nBuLi eq) THE m ” 3. Mel Me Me Me £P:CHO tuPo 4 te tro 4 AA ceanegcat HAAS yy fo ious6 on Nozake Kishi counting Reference: Anderson, J.C. Ley, S. Vs Marsden, $, P. Tetrahedron Let, 1994, 35, 2087. b Solution: ta, nul 1, BrCH,)sC1 MOH Tar” [Maes] THe 2.Mg, THE wt 9C3hy “SICH! Czy SE~(CH,) HICH). “B.oHG(O ECHO Aunt AAA AA rit OH, 1,0 oe A On Reference: Henrck,C.A. Tesrahedron 1977, 1848. Soluions to Chapter 7roblems * 161 ve ve « woe Khe wows pied SO ate wh woh OL a ons Amey rowers ection Reference: Kitagawa, Y.; Itoh, A. Hashimoto, ‘Am. Chem. Soc. 977, 99,3864, + Yamamoto, H.; Nozaki, H. J : on 7% : He 76 78°C OHO 20 - ‘4 ° mee ota Binh! 162 + Chapter 7 Formation of Caror-Carbon Bans Organometallic Reagents HO Nets Gum oh oh Step 1-2 Regiospecific ortholithation followed by boronic acid formation. Step 4 Suzuki coupling Step Benzylie deprotonation followed by intramolecular 1,2-addition and hydrolysis, Reference: Fu, JM; Sharp, M. J: Snieckus, V. Tetrahedron Lett. 1988, 29, 5459. 6. Retrosynthetic Analysis. Propose syntheses of the following compounds using a. Suzuki coupling procedure as a key step. Show (1) your retrosynthetic analysis and 2) all reagents and reaction conditions required to transform commercially available stating materials into the target ‘molecules. oon 3 ks a @ UY . PARRA act toto obtain Suva comtna bevcommin “sao Ko € jb a a wea = OCen Syuhesis: m4 Nt & o-pn 28 o> 22 WoC pean” OLD Patera et Naon Solution to Chaper 7 Problems + 163 5 stone on wine Se Se Rn 85% a a 4, TSOH (cat) Semen rol Reference: Soderquis, J. A; Rane, A.M. Tetrahedron Let. 1993, 34, 5031. inCata Retroxynthetic analysis: Me ‘Suzuki he a ve — ah ce MaHO03 ~e we wh, Sa NE 79% 98% 8) DIBALH, El 7 20 ayy 8 Por ERO one Br a 164 + Chapter7 Formation of Cabon-Cabon Bonds Vi Orgatonstali Reagents Me Me. 1a.LDA (1.05 20) BL THR o7eC er _2.PhyPoCHe er ‘nGeig 1b Gromige ° CH 16.1 20 0 Cota Catia XS RNMop Me. 30 888K, 30. PofPPhg) (eat) THE ™ 1GPO,, dioxane, 60°C IM Intact Su cup Reference: Miyaura, N.; Ishikawa, M.; Suzuki, A. Tetrahedron Lett 1992, 33, 2s CHAPTER 8 Formation of Carbon-Carbon n-Bonds Overview Chapter 8 focuses on the synthetic methods fo form carbon-carbon x bone. Problem | involves the applications of various reagents for carbon-carbon ‘bond formations. Problems 24 deal withthe selectivity issues associated with ‘carbon-carbon m-bond formation, The syntheses of TMs in Problem 5 require tilization of specific reagents to achieve chemo-, steeo-, or enantioselective ‘carbon-carbon m-bond formation. Key Concepts + B-Elimination reactions + Pyrolytic ym-elimination reactions of: ‘© Xanthates| © Selenoxides © Sulfoxides + Preparation of alkenes from alkynes ‘> Reduction (9 Hydrometalaion © Carbometalation ‘+ Preparation oF alkenes fom aldchydes/ketones ‘0 Wiig reaction ‘© Homer-Wadsworth-Fmmons (HWE) reaction ‘9 Peterson olefination ‘© Julis olefination Shapiro reaction Barton-MeCombie deoxygenation (Claisen rearrangement Cope rearrangement Preparation of alkynes ‘© Corey-Fuchs reaction Gilbert's reagent 16 + Chapter Fomation of Cabon-Cabon = Bonds SOLUTIONS TO CHAPTER 8 PROBLEMS ‘The more challenging problems are identified by an asterisk (*). 1. Reagents, Give the structure ofthe major product expected from each of, the following reactions. Be sue to indiate product stereochemistry. HO 1, ro yPcH,coset Nab THe aaNet A 2 Ho. 636 Pa (eat) EtORe at ‘Tes Solution a 0,8 2 SHO 1. e10),PoH:c0,e Nal TH Te80) eso * oe 5% 2 Ho, 5% PAC (cat) EtOAc eso" a 98% Step | Homer-Wadsworth-Emmons (HWE) olefination provides the (E)-vinyl ester Reference: Momén, E; Nicoleti, D.; Mout, A. J. Org. Chem. 2004, 69, 4615. 1. HaNNHTE, MeOH 22, r-BuL (2109) hxane, THE Sratto-20°¢ Boo, ~ 8 26.629 HOI workp Soltis fo Chater Problems 167 Solution: 1. HgAINHTS, MeOH 2a, Bul (21), hexane, THE 0 atta -20°C ‘Shapiro reaction 2.002 EB Peavervowe, oe 1a, NaH, OME, 0°C 18.C8, on 2 te. Met c 2. tlie, sened ube a 100" Solution: 4a, NaH, DME, 0° net a on as. eer cher H 4 4 f Xanthate 2. toluene sale tube ‘eos Vo A some G ‘Steps 1-2 Xanthate pyrolysis furnishes the alkene C via ym elimination, 1. TBSCL, imidazole, DMF 2a, TMSCHlaBr, Ei:0 OH 2 HOO 3. BANE, THF 168 + Chapter 8Fomaton of Carboo-Carbon «Bonds Solutio ° 9 oO 1. TBS, ini, OME orm 2a. TMSCH_MaBr, E10 Mass! oMgse 2b, HOzC-COgH, HzO. ‘Ores | 20. HOC-COWH, HO, oO + MosSi0-siMos bp= 101°C CH, 3. reBuyNF, THE > Step 2 Peterson olefination, on A AA oes oO” Cle Temoved by ‘deiaion 1. Phy, CBr, EN, CH:Cr2, 78°C tot 2a. DIBAL-H (ea), toluene, =78 °C 2b. Roche's al worn Ee 3. pBuNR THF me 4a. nBull 6 eq),hexane,—78"Cron 7% 40.2 HC! Solutio r 1. PPh, CBr EQN CHeble Br oe Aah EMH 8 co. comms acai 8 2a, DIBALH (39), toluone,-78°C 2b, Rochelle 2a workup br on & — oms Solution to Chapter 8 Problems * 169 3. mBUINE, THF, on e 71% Steps | & 4 Corey-Fuchs reaction. Step da Atleast 4 equivalents of n-BuLi are roqured for the transformation to proceed (2 x OH deprotonation, 1x Br elimination, 1 x acetylenic-Br lithium-halogen exchange), Step 4b Workup protocol Reference: Quéron, Es Let, R. Tetrahedron Let, 2004, 45,4527. 18, NaH (1.1 eq) DMF, 0° 18, PMB-CI 2. POC, NaOAe, Colle, CH rt on 2 PCC NaOAe, Colle, CHeOies gael sa conocer 4a, DISALH (2.1 09), CHCle 4, Roche's salt worup Solution: aeiatoas 28. PhyPaCHCOsEt, Colts, CHC 4a. DIBALH (21 69), Chee a pao ‘cHeoH PUB CORE a Ronoto eat or 4 woman e U ta. LDA (1.05 69), THF, -78 °C ae tr JR 0/00 eee) OO amaeee OK 2.” NaH, OME 170 + Chapter Fomution of Caton Cuban sands Solution: 48. LDA (1.08 eq), THF, 78° 1b Ot 0 or BAL PoVOMeI. £10" P(OVOMee o 10. NHC, acetone 2. NaH, DME CO ° 6 P(O\OMe)e 7494 Step Le Enol ether hydrolysis, Step2 Intramolecular Homer-Wedsworth-Fmmons (HWE) reaction furnishes the eyelc enone G. Reference: Piers, E.; Abeysekera, B. Scheffer, J. R. Tetrahedron Lett. 1979, a t ecioN chcHcO¥H ~Lores elas oo Le Sotto: we 98 a MeCiOMe)s, cHycH;Co,4 | EO? ~O 7 ~Lors ar sores ied ta oer ot oa io ‘oO SSL ores | aa ores nan 1 tat cot a Reference: Wipf, Ps Lim,S. J. Am. Chim. Soc. 1998, 117, $58. Soluion to Chap 8 Poblans © 171 Pr Sree a DIBAL-H (1.2 eq), Pr HO oe oreo -78-eo em workup 87% Were AA, x ‘ores Sep 3 HWE-modifcation of the Wittig reaction; teramethylguanidine is a mild, non-aucleophilie base (much like DBU) that deprotonates the ‘phosphonate reagent. Reference: Barrow, R.A; Hemscheidt, T; Liang, 1; Paik, S; Moore, R. Es TTius, M.A. JI Am, Chem Soe. 1998, 117, 2479. 42 9BULI (1.05 09), 18. | PhSCHL THF, 0° Te 78°C 1brdllag HCI workup 4b, PREHO AvadlaaHCiworkup yy A PHCHO ay 2 mOPGA (22 69), 6, Aez ore Chic, 0°C 5 ati), MeOH, EXOAC 8. POG, pyridine, 172 + Chapter 8 Fomaton of Caton.Carbon x Bonds Solution: 2, naPeA fo 1, PRSCHaLI]. oa 2200). oO THF.O°e (Ops GHe0in, 0°. om Wear Tpoo, wor pre 4a, n-BuLi (1.05 eq), MeO Ph Ph Tete 5. Naltio), Moor, y ea ‘sogph _E}0Ae 46 A00 2 om Step2 Oxidation tothe sulfone requires 2.09 mCPBA. Step3___Aleohol dehydration Steps 4-5 Julia olefination provides the (B)-olefin exclusively. Reference: Kocienski, P. J Lythgoe, Bs Ruston, $, J. Chem. Soe. Perkin Trans. I, 1978, 29. . OH 1 = “ ainaon THF cs ET a 2 QO g 003, aq best QO essen oO tee MeOz0" CO,Me MeO," "CO.Me Caen =n MeO2C" "come 48-2NNEOH, THE oc” *CO:H ae aq Ho Ks 6% Solions to Chaper 8 Problems © 173 5. 2qNaHCOs, Bry CHC Ka 7% Step 5 Halolactonization. Reference: Yoshino, T; Nagata, Y; lok, E.; Hashimoto, M. Katoh, Ts ‘Terashima, 8, Tetrahedron Lett, 1996, 37, 3475. ‘a, PhlgCt excess), THF, 78 °C 4b, sat NHC quench at-78 °C L 2. PreSlOCIOF Pe © ea) PreSOOGr Aha 86% Solution: 1a. PrWgCt (ess), THF, -7 c Hon “er 4b, sate NHC quench at 78 °C Sey sno no ben bn hominctas 2. PhgS|OCICFPhle (5 60) rece Opn L 86 ‘Step2 Alcohol dehydration with Ph;S[OC(CF,)sPhs (Martin sulfurane reagent) Reference: Boyd, V. As Drake, B. E; Sulikowshi, G. A. J. Org. Chem, 1993, 58,3191, 174 + Chapter 8 Formation of Cabon:Carbon x Bonds Selectivity. Show the reaction intermediates obtained after each step and the major product obtained as well as provide the reagents and conditions ‘when not given to accomplish the following transformations. i a. pBuli (1260) oo 2p 30. 61CO4Me aa ecco : “Ja Mezouui i ee@) Thitmte 9% f f , i setae, OTS a wxesen f f 9a, n-BuLi (1.2.€q) ‘COpMe . Steps 1-2 Conversion of a Ketone to a terminal alkyne (compate with Corey- Fuchs reaction where an aldehyde is converted toa terminal alkyne). Step 4 Me;CuLi adds tothe acetylenic ester ina conjugate manner to provide @rolefi, Reference: Marron, B. E Nicolaou, K.C. Synthesis 1989, $37, Sutions wo Chapters Problems * 175 ». 5 [Preis cone nat a 4 steps: a Nal, HMPA, 50°C Aten, MA Seg GaN THF, °C ie Le On (6b. Po(PPhg)4 (cal), refhux a sm Sotuto a 1. PhgP=CHOMe a 2. HOzC-COgH ‘CHO one THE O-Cton one HHO. SA oge vn acon «Tatars 04% 5 [rnoss come ua 3, NaBHy, E10H, 0°, a “.T8Gi, pyine ‘Nal BUPA,5O"C 69% $0,P0 62. Nal ‘COnMe THE, 0" _ ‘Se psermea 2596 salam fo. * Sospn ltused product 8 75% Reference: Trost, B. M, Verhoeven, T.R. J. Am. Chem. Soc. 1977, 9, 3867. > 1. (MoO },PCHN, KOFBU, THE, -78 °C LINGO) es ROFGY THE TBE ig Da DBALH 100), CHOin 7S aa, (tty, 25 Rochalla’ salt workup 176 + Chapters Formation of Catbon-Cabon sands Solution: 3 FO) 9 1 guonons norm, oy aoe rot > 2a, DIBAL-H 21 oq), CH,Cip,-78 2b, Roche's sa workup Step 1 Conversion ofan aldehyde tothe corresponding terminal alkyne with Gilbert's reagent Reference: Powell, N.A.; Roush, W.R. Org. Lett 2001, 3,453. 1. ChegBH (109), THE B.CHCO.D Oe See © NAOH HO a7 ‘Solution: way OMBH TH O-= Chg 4 b.cH:C0,0, 4 H NaOH, Hee . ot om Steps se (hydroboration-protonolysis-oxidation) represent a method for the selective reduction ofa terminal alkyne in the presence ofan alkene Reference: Zweilel, G.; Clark, G. M, Polston, N. L. J. Am. Chem, Soe. 1971, 93, 3395, Solutions to Chapter § Probie © 177 3. Stereochemistry. Predict the stereochemistry ofthe major product formed foreach ofthe following reactions. Give an explanation for your choice 1. NaBH, (excess), NCI MeOH 2-TSOH,aeetone, HO 3. (EIOYP(OICH:COEL NAA THF aoa (90:12) Solution: {Naot ron, Noieot” — CsHyCHOEe Ooty —==-CHE * oe 2 Teh, cone a (e0),P(0}0H,00%8, mo rot ono Kanne io a ea ea as con 40% (08:12 2642:2646 nintue) Step 3 HWE reaction. Reference: Byrne, B.; Lawter, LM. L Wengenroth, K.J. J. Org. Chem. 1986, 51,2601, os DMAP (cat) THF SL Bre we (04:4 mire of tomers) J 2. MeO, 60° 72% wo ea R= (CH,)08n 178 + Chapter 8 Formation of Cabon-Cabon Bonds ‘Solution 0 mom anna Fe) iO 2A Tad) ~ one ean oom neon oie = (CH2)g080 (P-keto ester formation y 15.dene @ cna tasot aes ED yyy 8 ow SAE wo Bg aad free, en OH ta nat so69, THF, 06 to Ave, QO 2&S5ncaer o (OS ZaPRP.DEAD PRCOSLTA Gone | HOOAO ate no BUSH Mors Hho 4200" Solution: ta.nat (19 on Tin oe QH aa. Pnye, DEAD, on tv, 180) PHO HE OD Boni Ho reso! a reso! co 80% Solutions to Chats 4 > 7 2 Aor, a 4.2000 HalOACe ‘Teso Taso. cz ao ‘Step. Mitsunobu reaction inverts the configuration of a2 alcohol Step 4 Claisen rearrangement. Reference: Clive, D. L. 1 Magnuson, S. 8; Manning, H. W.; Mayhew, D.L. J (Org. Chem. 1996, 61,2095. en 1.PhSeGi, CHa 78°C, ‘ores 2 MeO THF (tit: phenysetnolactonzation) Solution: 1. PhSeCl, CH,Cle 9 “78° ‘ores, Me, PHS Me one ofp. anoane |C ne 1 Reference: Curran, D. P Rakiewiez, D. M. Tetrahedron 1985, 41,3943, 180 + Chapter & Formation of Cathon-Caboa onde 4, Reactivity. Explain the repioseletivity and stereochemistry observed in cach ofthe following transformations. nad ES Cl, Solution Br ole BAL "ANF a sun, FSA b me 0 | a /=PPhs We, — A Oy wR |Z ( 05% Reference: Buchi, G; Pawlak, M. J. Org. Chem. 1975, 40, 100. 12, TMSCHaLI, EO 1b. HO workup CHO 2 GrOs pyridine Cont 3. UCHCO3t84) towne, exana, 0° se 4. HCIO,, THF ks Solon to Chaps robles © 18 ‘Solution: on fa. (TMSCHoLi E40 or 16.4.0 workup ow 2. 010s, pyri on ° 8. [LioH,003¢4) ‘imes euone, hexane, 0°C SiMe, - ‘cOntBu 4 Woo4 TH. 0°6 ote 20% Reference: Ruden, R.A; Gaffney, B.L. Synth. Commun. 1975, 5,15. = ta. n-Buli, THF, -78 °C 16. allyl bromige (0.5 eq) C ion Doon yeceost pea Saar 1 all bromide (0.5 ec) ~ a Dosm Fite [Psion] se o- ‘. tlturate with pentane ‘Son, DMSO, 7 ue - Step 1a ‘The enhanced acidity of cyclopropyl over normal secondary hydrogens permits the facile deprotonation using -BuLi; see J. Am. Chem. Soe 1973, 95, 3068, Step2 Methylation using magic methyl (caution: carcinogen!) Stcp3 Regioselective E2-elimination (no eyclopropene is formed due to ring strain). 182 + Chapter 8 Formation of Catbor-Carbon x Bonds Reference: Zuiterman, F.; Krief, A. J. Org. Chem, 1983, 48, 1135, 5 Synthesis. Supply the missing reagents required to accomplish each of the following syntheses. Be sure to contol the relative stereochemistry, p-p Solution: 1a, HyB-SMep, THF 1B, NaOH, Hee 2 Teoh pyidne 3 HBuGie HELO . F ' oadhet me onc Solution: fa. mBusCull, THF, -25°C 4 b.Phsoer, THE a on ome Solution to Chapter 8 Problems + 183 Reference: Mehta, G.; Krishnamurthy, N Karra,S.R. J. Am. Chem. Soc. 1991, Mies a ° Ou, ~ Qu Me Me Soto ° OL 1. NaBH tis O. 2. Fron Ch fe > Ha(OAc),. CHO Me Me 3 200%. Me 4a 4a 7 Mg Ot S.MnO2 ° “eo. HO ‘workup Referen Foran analogous synthesis, see Yamamoto, H.; Sham, H.L. J. Am Chem, Soc. 1979, 101, 1608. : mo whe ed : me Bee wae woe Bw RAL seer as oy ee, ene 2b. HF, HZ Reference: Normant, J. F Cabiez, G.; Chuit, C. Villers, J. J. Organomet (Chem. 1974, 77,281, 184 + Chapter8 Formation of Cabon.Cahon = Bonde f 1a cox ror Ho Soy Sotaton: omcio 122%. ogcyagy SORT ‘ HzO 2b. HG noe 20.H*, HzO ote wwe sertlooe we cose OAL gd AS Step 3 The stabilized ylide is generated i situ by the treatment ofthe precursor phosphonium salt with NaH. mmr, = COL ce 1 pon vo oe NR, SEBEL no as 7% 9. Ha, Lindlars cat ° ‘psOn MeOH on SPCC. CHC, — CT 5% 24 CO ‘COzH 95% Reference: Reyes, E. D.; Carballeira, NM. Synthesis 1997, 1195. Solution to Caper 8 Poblms © 188 bh 4 °: > NE one: # Solutio “1 PhP _ amore ofc tmeae ‘cvene schx 23 pine X Xe code) Cope earangenent aa o%0441.100) 4 + 240 pte (2:1), 70°C, : LOS eants05 wore Ta Nbtog frees TH, HO i y oe sa 0% (steps 1-2) Reference: Nozoe, 8. Tetrahedron Lett. 1976, 195, P F wT TEL wo P . . Ox “Banal 08,.THE OS HO" 2b. Mel 9 8. BuySH ves Ss aves, a sem wy a Steps2-3.Barton-MeCombie deoxygenation Reference: White, J.D. Jeffrey, .C. J. Org. Chem, 1996, 61,2600, CHAPTER 9 Syntheses of Carbocyclie Systems Overview Chapter 9 describes various methods for the synthesis of carbocyclic systems. The preparation of such systems can be achieved either by connecting {wo atoms within a single molecule (inramolecular reaction) of joining together two separate molecules (intermolecular reaction), Problem 1 focuses on reagents for the synthesis of carbocyclic systems Problems 2-4 address selectivity issues associated with carbocyelic systems. The syntheses of TMs in Problems 5 and 6 require the selection of specific reagents to achieve chemo-,stereo-, or enantioselective cyclization. Key Concepts ‘+ Intramolecular fee radical cyeizations 2 Acyloin condensation ‘9 Pinacol coupling, © McMurry reaction + Cation-n eyelizations + Perieyelic reactions © Diels-Alder reaction + Ring-closing olefin metathesis (RCM) © Grubbs catalyst 186 Solaonsa Capler9 Pblems © 187 SOLUTIONS TO CHAPTER 9 PROBLEMS ‘The more challenging problems are identified by an asterisk (*). 1. Reagents. Give the structure of the major product expected for each ofthe following reactions. Be sure to indicate’ product stereochemistry where applicable. ° 1.70458, CH, 40°C Q 21 POC, 4AMS, Gs OAL ROMO aa THF Orc sus Br 8 hele 9 woo AoA Pte spe ocho oS Frome, hs Q.naH THE OPO econ ‘0 GHC a Step 1 Lewis acid-mediated MOM deprotction Stcp3 Intramolecular HWE reaction. Reference: For an analogous reaction sequence, see Taillier, C.; Gille, B.; Bellosta, Vs Cossy, J. J. Org. Chem. 2008, 70, 2097. ». ° a. To Zn16u) JAA nov Oe Me —— THO work ¥ (Owain Hoy Solution ° 279 2409 SAA, ros “i t Mer et +b. HO workup “Bu teleryoonty > he Chapter 9 Sythe of Carey Systeme Reference: MeMunry, J. Kess, KL. J. Org. Chem. 1977, 42,2655, 19. LOA, THF, 70°C 7 sb TMC! “Patavene, 160°C eeaeaubey © Me 2b. an HCI workup Solution: ra.Loa tHe, -zare TS 2a. 160°C ems ~Teluene™ Me ramotcate iat Auerraacion 2.89 HOI Me o ce Reference: For an analogous synthesis, see Yamamoto, H.; Chem. Soe. 1979, 101, 1608. 13. LDA, THF, -76 1b, 2diroriogropene > oe “Ba MeLi (14.09), €10,-78°190°S gang 2b. 10% HCIO, work sham, H.L. J. Am. Sotto ° 1a. LDA, THF, ° oe oe Sad ass 2a ati (.98) ry = “10m 160, 190 NL tao “0 rm meg > Hon oR how0cy oud? 9.08 om Solution to Chaper 9 Problems + 189 Reference: Toyota, M; Hangovan, A.; Kashigawi, Y.; Ihara, M. Ong. Lett A. TisNHENHy, MeOH STE eon ene eee as L TMT 2a, s buts 00) u —o feo ra are a ae oan ate 25. #:O workup e oo Jones, T. K. Denmark, 8.B, Helv. Chim, Acta 1983, 66,2377 oe i Cif OrgPAueCHPh (cat) ay nt ey, bome Solution: te Cif OmsP-AUACHPh at) aw Sse 0 dose “ 4” "COMe snpcosrg meabess te F Reference: Ruljes, F. P. J. Ts Schoemaker, H. E. Tetrahedron Let, 1997, 38, on. 190 + Chapter 9 Syntheses of Carney Systems & 1. PhgPaCiMe}COZEt 4a, DIBALH (26 69) Heep rohox hoxane, CHaCip,~78 °C 2. POG, CHetle a1 2 Rochele’s sat workin oo PHP, CBry Zn, «86% Sa, mBULI oq), hexane, TH BEG Creat “78t00°C. 15. H,0 workup Solution: 1. PhyPoiMe}CO2Et 49. DIBALH (25 eq) CHeCle rahe hexane, CHCl 2. Pot, CHscle & Me 278°C. 8. PhP, CBr, Zn, 4b. Rocholl’s sat CHC BG, Coxeter 20% 5a. n-BuLi (4 5b. 20 foxanes THe work, er Me | ~7e" 90°C er om OF Step 1 Wittig olefination gives the (E}-alkene as the major product (93 : 7). Step 5 Corey-Fuchs alkynylation. Reference: Oppolzer, Ws Robyr,C. Tetrahedron 1994, 50,415. : seteey Ed oH, SE Mo’ aa ero rm = fF 4 om Reference: Blanchard, A. N; Bumel, DJ. Tetrahedron Lett 2001, 42,4719. Solitons to Chapter9 Problems * 191 La (©-0H,CH-CHCHO, NHCOBn 110°C 76% {soaled ampule) Solution: Ce Le (@-0HsoH-cHCHO. “CHO NHCOBn 1070 (sealed ampule) 1 70% ‘Overman, L. Jessup, PJ. J Am, Chem, Soe. 1978, 100, 5179, Toon (eat) Toone, ree gan, A Io ue -Ht ° Jolene — DewrAer eZ ee |Y [2 eee as Ma} Te aay ; pronation Reference: Kodama, M.; Kurihara, T. Sasaki, J Ito, S. Can. J. Chem, 1979, 57,3343, 102 + Chapter 9 Syntheses of Carocyeli Systems 2. Selectivity. Show the product(s) obained and the appropriate reagent(s) necessary foreach ofthe following transformations. me tS Benzene, 100°C (ele tbo) os 2a. NaOH, HO, doxane, 40°C hay ¥ 25.09 HCiworkap Solution: ° Me A Meo Derzene, heat Myo” ° ° 2a. NaOH, HO Ho soxane, 40°C Bb. AGO! WORD tygq 4 te A ode Slop 1 The more electron rich, methoxy-substituted double bond is less ionophilic than the methyl-substituted one; hence the cycloaddition ‘occurs selectively atthe methyl-substtuted double bond. Referen Woodward, RB; Sondheimer, F; Taub, D; Heusler, Ks MeLamore, W.M. J. Am. Chem. Soc. 1982, 74, 4223, Solution to Caper 9 Problems + 193 Meocly 1. Nay MeOH. Te OH mo: ‘MOM 2 MONG Eu + oH:C, Fg ycOeMe 72% eh aap titan 8 90% Reference: Sieburth, &.M; Santos, ED. Tetrahedron Lett, 1994, 35, 8127 come 3.710 stops 2A) 2st zt g eat 5% ‘Solution: 14.05, MoOH, 35°C tb. Mees 2: NaOMe, MeOH, eh Me? 8. Zn(A9) ‘DME, heat a Meo c 50% ‘Step 2. Epimerization of tothe more stable C()-Hy isomer. Step 3 Intramolecular MeMurry coupling 194 + Chapter Syesss of Cubocyei Systems Reference: Mikami, K.; Takahashi, K.; Nakai, T. J. Am, Chem, Soc. 1990, 112, hot. cla” em CGE rot gh She, Fm fandom MeO muy sngetosng motto ite fe ier Moos yO La [Ru=CM Ne trurowes OY” . 2% Reference: Boyer, FD, Hanna, 1 Ricard, L. Org. Lett 2004, 6, 1817, Solutions to Capt 9 Poblams + 198 4 MOM OH 4. crgC(OE Ng we coe oe AA Rage woo CL “ase ‘ coset 2. ashy oe socom, oS BE wMa0zeSA~ praror eo meh e eee ee sbimon oo oer isbonnarmpee COzE 9.15 PP imidazole 4 + TOADS, THE OS t Slep2 Chemoselective reduction ofthe c-alkoxy ester. Step 3 Ester enolate alkylation (5-exo-tet cyclization), Reference: Kim, D.; Lee, J5 Shim, P. 1; Lim, J. Ls Jo, Hs Kim, 8. J. Ong Chem. 2002, 67, 764 3. Stereochemistry. Predict the stereochemistry ofthe major produet formed foreach ofthe following reactions. 1. AFoms Bn GOaMe 360° (eed ube) pmtesine este tbe A Me 2 BF, OHON, oy 8. PDC, 4Amol, ives, CHCle 196 +08 lr 9 Syrihesesof Carbocele Stems |8.P00.4 Amol ives ot Reference: Kan, T.; Hosokawa, S.; Nara, S; Oikawa, M.; Ito, $s Matsuda, Fi; Shirahama, H. J. Org. Chem. 1994, 59, $532. , aa Gm * = (sealed tube) oe 9 . s90"0 Me ‘olvene 4. (seated tbe) © consider the polarization ofa ving group Endost (potarty mismatch) Reference: Ireland, R. E.; Thompson, W. 1. J. Org. Chem. 1979, 4, 3583 ° ° @ LIN(TMS)2, TMSCI gos One SS Teeagnowomm Sy ord SEM = CH_CH,SIMes Solitons to Chapter Problems * 197 Solution LNT) TSI ° ° fosem "THF, 100"C 4 oo eerie oN Posen pret gs Se tame SN ovo oF 0 SEM =CH:CHaSittog © 17196 (20:1 0) Step a Silylation ofthe ester enolate to give a silyl ketene acetal Step b (3,3) sigmatropic shift (ester enolate Claisen rearrangement) via a chair ‘Gansition state on the sterically more accessible P-face of the butenolide Below. Lose ows yrs Yves — mie 2 H chair TS-A(cstavored) chal TS-8 (fevered) Reference: Burke, . D.; Pacofsky, G. J, Tetrahedron Lett 1986, 27,44. ws, |ogk Mo, ‘4 He Palo oH Hor EIOn yee wie > 198 + Chapter Sythe of Croeyei Systems This is one of the finest single-step transformations in the chemical literature. [Note that five reactions are achieved in one pot: hydrogenation of two double bonds, Chi deprotection, intramolecular imine formation, and convex-face alkene hydrogenation that establishes the n-Prsterocenter. Reference: Overman; L. E. Jessup, P.J. J. Am. Chem Soc. 1978, 100, 5179. ~ srogPh Me Me a. BF:OFtp,CHeCip,-78 °C ag NaHCO workup a. BFOEe CHCl, 78°C ‘Remote aba enruro on Fs, wile Po wot Ro chy Ly 0g, NaHCO BoE rot RE Chitin 0-036 of 44 2cycbadaton 0% endo select Steps 1-2. See Tetrahedron 2000, 56,2195, Step 4 Swern oxidation resulted in aromatization ofthe 1 4-quinone. Reference: Lee, W-D.; Kim, Ks Sulikowski, G. A. Ong. Lett. 2008, 7, 1687, : 0 = 0: O= = AUS cin ° oe SHO sa Ung THE 3.0s, CHCl, -78°C_ OKC Oo 5, HY, HzO He 76 Lunt, THE ft ag ma nef Se Hee Hy ‘cree CH Hae Solon to Chaper 9 Problems © 207 6. Retrosynthetic Analysis. a. Propose a synthesis of (-}-conicene using a ring-closing metathesis (RCM) procedure as a key step, Show (1) your retrosynthetc analysis and (2) all reagents and reaction conditions required to transform a commercially available starting material othe target molecule S (peonicene Retrosynthetic analysis: - Ses Bs or > 6 wv oN a Synthesis: 41. 80040, NaOH, dioxane Once mictasos.vr Chor rmbssteTHe OOH Ga, DIBALA (Tt ea) WoTCHO 5 Tow HOI Meo 4 Boone, ce boo (Seecontecten) Prone % Rodel a wortap « ficetrorncl, om bee. tH cme nc 1 GRP, (005 0 Ov “ote? : ome ca. ta os wy i A EEO oy Taher, 18h @ ca Lai Tar I AN ° 86. NaoH Ho se con = Ye Reference: Arisawa, Mj Takezawa, E.; Nishida, A Mori, M; Nakagawa, M. Synlett 1997, 1179, 208 + Chapter 9 Syntheses of Cabocyelie Syms 'b. Propose syntheses of pumiliotoxin C using (1) an intramolecular Diels- Alder cycloaddition asa key step and (2) via a conjugate addition ofa side hain, ether the C(5)-methyl or Cc2-n-propylgx0Up. Ch wh puriotoxin C (1) Intramolecular Diels-Alder cycloaddition approach ASDA NW N 4, ‘Sols Aer oacion Saree ' cea on Symtess: “~y Me Acno coo — morose 20'0.200 HATS Ziecoc — ~~ toluene a -EtgN, CHeClz MOA A 8. (erystaization) a Ast Pa MeOH poe SPT AN 5mm Solutions to Chaper 9 Problems + 208 Reference: Oppolzer, W. Flaskamp, E; Bieber, L. W. Helv. Chim. deta 2001, 84,141 (@) Conjugate addition of the C¢s}-methyl group Retrosynthetic analysis: - Ml —- AD.. af CHePh (aap) hae 2 Stereoselective methyl addition oceurs from the ‘convex’ fae (be 3°-amide is inert to the cuprate reagent) ‘Reference: Ibuka, 7; Inubushi, ¥ Lett 1978, 323, 5 Saji Ls Tanaka, K; Masaki, N, Tetrahedron 210 + Chapter Syntoses of Cubosysi Systems (@ Conjugate addition of the C2-n-propyl group Retrosynthetic analysis: Synutesis: gee gee C O 1. mPeig cu C QO 2, Nats, 0804 (ca) Aye BREET gay coxph cop ow 9 \, 9 3 4a, MopCuli, ak C oO BF EtO AO. by? nee bon boxe 0% ors Soltons to Chaper9 Problems © 211 5a, LDA, THE 62. Hs PtOe 3b. PANT ‘npr 6-H! op aye wy 6o,Ph = Ho! orm Step2. Lemieux-Jolnson reaction. Step 6a Catalytic hydrogenation reduces the vinyl triflate moiety and cleaves the (Coy protecting group. Reference: Comins, D. L; Dehghani, A. J. Chem. Soc., Chem. Commun, 1993, 1838,

You might also like